diff --git "a/validation.csv" "b/validation.csv" --- "a/validation.csv" +++ "b/validation.csv" @@ -1,31 +1,265 @@ id,question,opa,opb,opc,opd,cop,choice_type,exp,subject_name,topic_name -f94f9f8f-fb28-44f0-9381-3dca473e305d,Atherosclerosis initiation by fibroblast plaque is mediated by injury to ?,Smooth muscle,Media,Adventitia,Endothelium,3,single,"Ans. is 'd' i.e., Endothelium The most acceptable hypothesis for the pathogenesis of atherosclerosis is ""the response to injury hypothesis"". According to this hypothesis, atherosclerosis is a chronic inflammatory response of the aerial wall initiated by injury to endothelium. Pathogenesis of atherosclerosis Following stages occurs in the pathogenesis of Atherosclerosis: Endothelial injury Earliest stages of the development of atherosclerosis are mediated by the inflammatory cascade. Inflammation mediated injury to endothelium is the cornestone in the development of atherosclerosis. After injury, endothelium is activated and there is increased expression of adhesion molecule-VCAM-1 and there is increased permeability to endothelium. TNF is the major cytokine to induce this expression. Migration of leukocytes When VCAM-1 is expressed on endothelium, leukocytes adhere to the endothelium. Leukocytes than cross the endothelial barrier and begin to accumulate in subendothelial intimal space. Macrophages engulf LDL cholesterol and form foam cells - formation of earliest lesion, i.e. fatty streak. Macrophages also form oxygen free radicals that cause oxidation of LDL to yield oxidized LDL (modified LDL). Smooth muscle cell migration and proliferation Inflammatory cells in subendothelial intimal space secrete cytokines, mainly PDGF, TGF-ct and FGF which cause migration of smooth muscle cells from media to subendothelial intimal space as well as their proliferation. Maturation of plaque Smooth muscle cells synthesize extracellular matrix (especially collegen) and conve a fatty streak into a mature fibrofatty atheroma, and contribute to the progressive growth of atherosclerotic lesions.",Pathology, -bdf94942-49c0-4d9a-b990-a7bfb7099857,The WBC that is considered “second line of defence” is:,Neutrophil,Eosinophil,Basophil,Monocyte,3,single,"Monocyte is considered as the second line of defence. -  -Phagocytosis by Macrophages. -Macrophages are the end-stage product of monocytes that enter the tissues from the blood. When activated by the immune system, they are much more powerful phagocytes than neutrophils, often capable of phagocytizing as many as 100 bacteria. They also have the ability to engulf much larger particles, even whole RBCs or, occasionally, malarial parasites, whereas neutrophils are not capable of phagocytizing particles much larger than bacteria. Also, after digesting particles, macrophages can extrude the residual products and often survive and function for many more months. -Reference: GUYTON AND HALL TEXTBOOK OF MEDICAL PHYSIOLOGY, THIRTEENTH EDITION(INTERNATIONAL EDITION ) page no 458",Physiology, -cbc8a1b1-532c-490e-a17d-9168083fabae,Mark the true statement regarding use of Lithium in maniac-depressive psychosis?,Monotherapy for acute episodes,Monitoring of serum concentration is not useful for guiding dose adjustment,Can result in leucocytosis,Can result in hypehyroidism on long term use,2,multi,"Lithium Use : Prophylaxis of mania alone, with sedatives in acute mania. Monitoring of S. concentration is significant to know the toxicity of drug S/E of Lithium Lithium use for a long time causes benign and reversible T-wave flattening Seizures Polyuria and compensatory polydipsia Dermatitis, folliculitis, vasculitis Weight gain",Pharmacology,Psychiatric Illness -2b7ff41e-66e4-467a-97ec-9ea4e5e98cbc,Determination of Pulp vitality by pulse oximeter based on:,Beer's law.,Pascal's law.,Doppler law.,Poisslues law.,0,single,"Pulse oxymetry- - -Based on Beer Lambert law -Utilization of a probe that transmits red (640 mm) and infrared light -Photo detector as a receptor site -Different absorption ability of light by oxygenated and deoxygenated haemoglobin leads to changes in the amount of light differently absorbed by the vascular bed before reaching the photo detector.",Dental, -c5165fb3-3a59-49ed-9de0-4dbbced2cf08,An Isograft indicates transfer of tissues between -,Unrelated donors,Related donors,Monozygotic twins,From the same individual,2,single,,Surgery, -e197f947-1bdd-4efd-887a-b72faa9441c5,All of the following are causes of congenital blindness Except,Toxoplasmosis,Congenital rubella,Vitamin A deficiency,Cataract,2,multi,"Causes of congenital blindness:ToxoplasmosisCongenital cataractsGenetic diseaseCongenital rubellaCongenital glaucomaUsher syndrome(Refer: Nelson's Textbook of Pediatrics, SAE, 1st edition, pg no. 318 - 319)",Pediatrics,All India exam -b15c5039-0f3c-4195-8331-35b9632899c7,Hypersecretory glaucoma is seen in –,Epidemic dropsy,Marfan's syndrome,Hypertension,Diabetes,0,single,"""In epidemic dropsy, glaucoma is wide angle glaucoma, with normal chamber and angle. Outflow is within normal limits. The queous shows raised level of histamin, prostaglandin and protein suggesting that glaucoma is hypersecretory"". _ Mukherjee",Ophthalmology, -01b9f068-4db2-45e5-94b8-19d4f7d290f7,"A 14-year-old girl presents with quadriparesis, facial palsy, winging of scapula and ptosis. There is h/o similar illness in father and brother but less severe. Her CPK level is also raised (500IU/L). She is probably suffering from?",Emery-Dreifuss muscular dystrophy,Becker muscular dystrophy,Limb-girdle dystrophy,Scapulofaciohumeral dystrophy,3,single,"Facial muscle involvement is not seen with Becker and Limb-girdle muscular dystrophy Emery Dreifuss and Becker are XLR disease so can't be present in a girl child. Hence by exclusion the diagnosis is Dx: Scapulo-Faciohumeraldystrophy. Facio-Scapulo-humeral dystrophy Autosomal dominant inheritance and has an onset in childhood or young adulthood. Facial weakness is the initial manifestation-->inability to smile, whistle or fully close the eyes. Weakness of the shoulder girdles-->makes arm elevation difficult. Scapular winging becomes apparent with attempts at abduction and forward movement of the arms. The serum CK level may be normal or mildly elevated. No specific treatment is available, ankle-foot ohoses are helpful for foot-drop.",Medicine,Myasthenia Gravis and Muscular dystrophy -44e3f7ec-ad90-49bc-9f91-54f959bae81b,Which of the following is true statement regarding human eye,Cornea cut off wavelength upto 400 nm,Normal eye medium will permit wavelength of 400- 700 nm,Even after cataract surgery UV rays are not penetrated,Lens will not reflect light,1,multi,"Ans. b (normal rye medium will permit wavelength of 400- 700 ntn) Ref: yanoff 3rd ed p 383, internet sitesCornea and lens act as special filters absorbing the more energetic wavelength of the electromagnetic spectrum that can damage the retina. Cornea absorbs wavelength below 295nm where as lens absorbs strongly in the long U V B (300 - 315) and most of the UV A (315 - 400) wave-lengths. Eye light transmittance is relatively high in the 400nm-700nm range.",Ophthalmology,Anatomy -6a19df3c-6af0-4673-8e2b-02d86ddc7523,Which of the following muscle is not a hybrid muscle,Pectoralis major,Extensor digitorum,Flexor digitorum profundus,Brachialis,1,single,"Extensor digitorum Hybrid muscles are the muscles having more than one nerve supply Pctoralis major: lateral & medial pectoral nerves Brachialis: additional proprioceptive fibres from radial nerve, in addition to its muscular nerve, musculocutaneous nerve. Extensor digitorum is supplied by the posterior interosseous nerve only.",Surgery, -161fc4a5-e9fb-44c6-9a4b-2d734cf01ebc,Delayed union of fracture of a bone follo-wing a surgical treatment may be due to,Infection,Inadequate circulation,Inadequate mobilization,All of the above,3,multi,D i.e. All,Surgery, -f18b19db-efc0-40d3-accd-5d2dcb3b1879,What is the pressure of inspiration?,Intrapleuarl,Transpulmonary,Trans chest wall,Alveolar pressure,0,multi,"Ans: a (Intrapleural) Ref: Ganong,22nd ed,p. 6651The main pressure in lung related to inspiration is intrapleural pressure.Intrapleural pressure.The pressure in the intrapleural space between the lungs and chest wall is intrapleural pressure.Intrapleural pressure changes from -2.5 to -6 mmHg during inspiration. During strong inspiratory effort it may reach up to -30 mmHg.Transpulmonary pressureIt is the difference between alveolar pressure and intrapleural pressure. It is the measure of elastic forces in the lungs.Alveolar pressure or intrapulmonary pressureIt is the pressure inside the lung alveoli. When no air flows the alveolar pressure is considered to be equal to the atmospheric pressure, i.e., 0 cm of water.During inspiration it decreases to -1 cm of water.During expiration it increases to+1 cm of water.",Physiology,Respiratory System -cdfbd205-6f98-4cdc-9d76-29c0e9887cef,One standard detion in normal standard curev includes value-,50%,68%,95%,100%,1,single,Ref:Parks 23rd edition pg 849. 1SD : confidence limit = 68% 2SD : confidence limit = 95% 3SD : confidence limit = 99.7%,Social & Preventive Medicine,Biostatistics -d000623f-eba6-4e64-8cc0-3ed932403fce,Foam cells are seen in infection with which virus ?,Measles,EBV,Molluscum contagiosum,RSV,1,single,"Ans. is 'b' i.e., EBV EBV encephalomyelitis shows :Abnormal white matter with perivascular infiltrates of inflammatory cells and foam cells.Some myelin is replaced by lipid-filled macrophages and hyperplastic astrocytes.",Microbiology, -fc374693-12de-4ca4-8854-34bd1a9c686b,"During extraction of the upper first molar, the mesio buccal root is missing and is suspected to have been pushed into the maxillary sinus. -The best position for the patient immediately after the incident is",To position the patient upright,To position the patients in a semi inclined posture to visualize the perforation,Trendelenberg position,Reverse trendelenberg position,0,single,,Surgery, -ffa5b47b-7d02-489f-9bcf-f6753b2be449,Rescue P.C.I is done for which of the following?,Persistent chest pain with ST elevation > 60 min after thrombolysis,Persistent chest pain with ST elevation > 30 min after thrombolysis,Persistent Chest pain with ST elevation >90 min after thrombolysis,Pain with ST elevation for >120minutes after thrombolysis,2,single,Option C = Rescue P.C.I. After thrombolysis ST Elevation begin to reduce If ST Elevation persisting in spite of thrombolysis. So it's failure of thrombolysis | within 90 minutes Rescue PCI Types of PCI Primary PCI done for STEMI (Door to device time < 90 minutes). Delayed PCI done for Unstable Angina NSTEMI Rescue PCI Failure of thrombolysis (within 90 minutes).,Medicine,Acute coronary syndrome -fefb2a6d-129a-4ebd-9100-fcdb862c98a2,Fever associated with infection,Accelerates bacterial and viral replication,Left shifts the oxygen dissociation curve,Is due to interleukin-1 released from CD4 T-helper cells,Increase oxygen release to tissue,3,single,"Fever right shifts the oxygen dissociation curve, which allows for a greater release of oxygen to the tissue, making oxygen easily available to neutrophils and monocytes for use in the oxygen-dependent myeloperoxidase system of bactericidal killing. Having more oxygen available to tissue also enhances wound healing. In addition, fever interferes with bacterial and viral replication by providing a hostile environment for incubation. Fever is produced by the release of interleukin-I from macrophages. Interleukin-I, in turn, stimulates the hypothalamus to synthesize prostaglandins, which interact with the thermoregulatory center to produce fever.",Physiology,General physiology -8cf45c14-9de1-4f72-a51c-e0ab03cd4518,Last organ to putrefy in females is?,Liver,Uterus,Brain,Breast,1,single,"Ans. is 'b' i.e., Uterus Order of putrefactiono Order of appearance of putrefaction from earliest to last is : larynx, trachea > stomach, intestine > spleen, liver > brain, lungs > heart > kidney, bladder > uterus/prostate > bone.o Amongst the soft tissues uterus in female and prostate in malesm are last to undergo putrefaction, as uterus and prostate resist putrefaction. Otherwise bones are last to undergo putrefaction, overall.",Forensic Medicine,Injuries -567e1d6a-5444-4e1a-b07a-98417b051c28,Hamartomatous lung tissue is?,Hypoplasia of lung,Congenital cyst,Lobar sequestration,Congenital cystic adenomatoid malformation,3,single,"Ans. (d) Congenital cystic adenomatoid malformation(Ref: Robbins 9th/pg 670; 8th/pg 679)Congenital cystic adenomatoid malformation (CCAM): Hamartomatous Q or dysplastic lung tissue, usually confined to one lobe.",Pathology,Respiration -c50f51df-27d7-43e6-9be6-4b89df294edb,Test for diagnosis of pyogenic meningitis is?,Widal,CSF PCR,CSF examination,PET scan,2,single,ANSWER: (C) CSF examinationREF: Harrison 17th ed chapter 376The diagnosis of bacterial meningitis is made by examination of the CSF. The need to obtain neuroimaging studies (CT or MRI) prior to LP requires clinical judgment.A broad-range PCR can detect small numbers of viable and nonviable organisms in CSF and is expected to be useful for making a diagnosis of bacterial meningitis in patients who have been pretreated with oral or parenteral antibiotics and in whom Gram's stain and CSF culture are negative,Microbiology,Bacteria +fabba8e7-51aa-4705-90cf-b310d23a59a7,True about the statement is: When null hypothesis is rejected when it is true?,Type I alpha error,Type II beta error,Gamma error,All of the above,0,multi,,Dental, +3118ef6a-713b-4bc5-908e-6bab3766e8bc,Thin filament consists of all except:,Actin,Troponin,Myosin,Tropomyosin,2,multi,Myosin is thick filament. All others  in the options are thin filaments.,Physiology, +43746273-1b76-4c58-97ac-7de0af6fdab0,"A 1-year-old boy presents with a delay in motor development. Progressive muscle weakness and blindness ensue, and the patient dies within a year. The brain at autopsy shows swollen neurons that contain numerous lysosomes filled with lipid. Which of the following is the most likely diagnosis?",AL amyloidosis,Hurler syndrome,Phenylketonuria,Tay-Sachs disease,3,single,"Tay-Sachs disease is a lethal, autosomal recessive disorder caused by an inborn deficiency of hexosaminidase A, which permits the accumulation of ganglioside GM1 in CNS neurons. The disease is fatal in infancy and early childhood. Retinal involvement increases macular transparency and is responsible for a cherry-red spot in the macula. On histologic examination, lipid droplets are seen in the cytoplasm of distended nerve cells of the CNS and peripheral nervous system. Electron microscopy reveals the lipid within lysosomes in the form of whorled ""myelin figures."" Swollen neurons that exhibit marked vacuolization of the perikaryon and contain lysosomes filled with lipid can also occur in other lipid-storage diseases (e.g., Gaucher disease, Niemann-Pick disease). The other diseases do not produce such neuronal changes.Diagnosis: Tay-Sachs disease",Pathology,Central Nervous System +3589876c-919b-4ce7-ba8b-598a71343f7d,Most common cause of neonatal meningitis-,Staphylococcus,E. coli,H. influenze,Pneumococcus,1,single,"Ans. is 'b' i.e., E. coli o E. coli & streptococcus agalactie (group B streptococci) are the two most common cause of neonatal sepsis and meningitis.",Pediatrics, +fa080eec-d8c0-4bbc-9d03-14288e46a80e,Primary complex in which of the following sites suggest congenital tuberculosis -,Lungs,Liver,Lymph nodes,Skin,1,single,,Medicine, +69f8092f-1fd4-489f-818f-72c1bd562463,R-factor in bacteria is transferred by,Transduction,Transformation,Conjugation,Vertical transmission,2,single,"Conjugation + +Bacterial conjugation is the transfer of genetic material between bacteria through direct cell to cell contact or through a bridge- like the connection between two cells. +Conjugation is a process whereby a donor (male) bacterium makes physical contact with a recipient (female) bacterium. +Donor status is determined by the presence of plasmid. +This plasmid codes for specialized fimbria (sex pilus) and for self-transfer. +Sex pilus (conjugation tube) helps in transfer of genetic material from male bacterium to female bacterium. +The plasmid is known as transfer factor (sex factor or fertility factor). +The plasmid may be R factory which codes for transferrable multiple drug resistance. +The DNA of the plasmid replicates during transfer so that each bacterium receives a copy → Recipient becomes a donor and the donor retains its donor status.",Microbiology, +21ec65bc-a0f9-4c62-b8b0-0d3dc5f8b331,CO2 is primarily transpoed in the aerial blood as,Dissolved CO2.,Carbonic Acid.,Carbamino-hemoglobin.,Bicarbonate,3,single,"D i.e. Bicarbonate Transpo of CO2 in the form of bicarbonate ions accounts for approximately 70% of transpoed carbon dioxide from the tissues to lungs Q. So CO2 is transpoed as plasma HCO3 - > RBC > HCO3- > Carbamino compound > dissolved CO2 Q &Transpo of Carbon Dioxide Carbon dioxide is transpoed in the blood as: HCO3- (70%) > carbamino compounds (23%) > dissolved CO2 (7%) Chloride Shift/ Hamburger Phenomenon When the negatively charged (HCO3-) bicarbonate ions move out of red blood cell into the plasma, to maintain the electrolytic equilibrium the negatively charged chloride ions move into the Red blood Cells from plasma (In plasma plenty of sodium chloride is present). This process is mediated by band 3 membrane protein Chloride shift occurs in: 1 secondQ Hematocrit of venous blood is: greater (by 3%)than that of aerial bloodQ PH of venous blood (7.36) is lower than that of aerial blood (7.40)Q * For each CO2 molecule added to RBC there is increase in one osmotically active paicle in cell either HCO3- or CI-. So RBC take up water & swell. For this reason plus a fact that some aerial fluid returns lymphatics rather than the veins, the hematocrit of venous blood is 3% greater than aerial bloodQ. Amount of 02 enters the body/min = 250 inVininQ Amount of CO2 excreted by lung/ min = 200 inVininQ (288 L/day)Q * CO2 is 20 times more soluble than 02",Physiology, +f09c1184-52d9-481d-97be-018d1a798945,What is the sequence of development of pubey in girls ?,"Thelarche, Pubarche, linear growth spu, Menarche,","Pubarche, Thelarche, Menarche, linear growth spu","Pubarche, Menarche, Thelarche, linear growth spu","Menarche, Thelarche, Pubarche, linear growth spu",0,single,"The sequence of SPECIFIC events of Pubeal events in girls is thelarche, pubarche, linear growth spu, menarche. Please note: This is not to confuse with accelarated general physical growth which is the first event in pubey Pubey Specific events in girls Events in boys 1st Thelarche Testicular enlargement 2nd Pubarche Penile enlargement & pubic hairs 3rd Linear Growth,Height, spu Linear Growth spu 4th Menarche Axillary hair and facial hairs In girls, pubeal development typically takes place over 4.5 years. The first sign of pubey is accelerated growth, and breast budding is usually the first recognized pubeal change, followed by the appearance of pubic hair, peakgrowth velocity, and menarche.... Novaks Gyne",Gynaecology & Obstetrics,DNB 2018 +5fb4705e-3b44-41da-ad08-25688404bdab,"A 70 - year- old man was administered penicilline intravenously. Within 5 minutes, he developed genralised urticaria, swelling of lips, hypotension and bronchospasm. The first choice of treatment is to administer:",chlorphenitramine inj.,Epinephrine inj.,High dose hydrocortisone tablet,nebulised salbutamol,1,single,"Ans. is 'b' i.e. epinephrine injection Urticaria, swelling of lips, hypotension and bronchospasm within 5 minutes of i.v penicillin suggests anaphylactic reaction. It's a major problem with the use of penicillin.The t/t of choice for anaphylaxis is i.v epinephrine.",Pharmacology,Penicillin & Cephalosporin +663ce243-7b00-468a-b2cb-aea89130cc9b,"Daily water requirement in child weighing 30 kgs, height 123 m and BSA of 1 m2 is-",1300 m1,1700 ml,2000 ml,2500 ml,1,single,"Ans. is 'b' i.e., 1700 ml Fluid requirment in a child o It depends on the weight of a child :- Weight Water requirments < 10 kg o 100 ml/kg/day 10-20 kg o 100 ml / kg / day for first 10 kg (i.e. 1000 ml) + 50 ml / kg / d for every kg above 10 kg. >20 kg o 100 ml/kg/day for first 10 kg (i.e. 1000 ml) + 50 ml / kg / d for next 10 kg (i.e. 500 ml) + 20 ml / kg / d for every kg above 20 kg. o The child in question has weight of 30 kg. o So, fluid requirment will be :- i) 1000 ml for first 10 kg Plus ii) 500 ml for 10-20 kg Plus iii) 200 ml (20 x 10) for 20-30 kg o Total requirment will be 1700 ml.",Pediatrics, +68f71e37-a2a5-44ef-9139-71914b863fa4,Drug of choice for ADHD is,Methylphenidate,Modafinil,Amitriptyline,Adrenaline,0,single,.,Anatomy,General anatomy +07869051-9b4b-47c8-8f93-1b9828078a28,Double bubble sign is seen in -,Ladd's band,Annular pancreas,Duodenal atresia,All of the above,3,multi,"Ans. is 'd' i.e., All of the above o Double bubble sign is seen in duodenal atresia, duodenal web, deudenal stenosis, Ladd's band, Annular pancreas, Malrotation of gut, preduodenal vein.Single Bubble signCongenita] hypertrophic pyloric stenosisDouble Bubble signDuodenal atresiaDuodenal webDuodena] stenosisLadd's bandAnnular pancreasMalrotation of gutPreduodenal veinTriple Bubble signJejunal or proximal ileum atresiaMultiple Bubble signIleal atresia",Radiology,Abdominal Radiography +b9aac52e-05c8-4feb-9869-ab57c9704bdd,A 16 year old boy does not attend school because of the fear of being harmed by school mates. He thinks that his classmates laugh at him and talk about him. He is even scared of going out of the market. He is most likely suffering from:,Anxiety disorder,Manic depressive psychosis (bipolar disorder),Adjustment reaction,Schizophrenia,3,single,"The history is suggestive of delusion of persecution (fear that schoolmates may ""harm"" him) and delusion of reference (belief that classmates laugh at him and talk about him). Had they not used the term ""harm"" and ""scared"" a diagnosis of social anxiety disorder could have been enteained.",Psychiatry,Schizophrenia Spectrum and Other Psychotic Disorders +abd192eb-0adf-49cf-ac78-e01855cfbce7,Bad trip is seen with which of the following drugs?,Cannabis,LSD,None,Both,1,multi,LSD is a type of hallicinogen LSD means lysergic acid diethylamide LSD was introduced and first used by ALBE HOFFMANNBad trip is seen with LSD and cannabis but common with LSD Ref: Essentials of postgraduate psychiatry By JN Vyas 1st ed Pg 330-336,Psychiatry,Substance abuse +9efce15d-8682-4927-b158-6927e4fd276e,Chromosome involved in Wilm's tumor is,11,13,18,22,0,single,WTl is the best characterized Wilms tumor gene. It is located at chromosome llp13 and encodes for a transcription factor that is critical for normal development of kidneys and gonads. WT2 is localized to a cluster of genes at llpl5. Reference: Essential paediatrics; O.P.Ghai; Childhood Malignancies; Page no: 617,Pediatrics,Childhood tumors +79efe49b-5816-4588-a0d7-4f9433db2335,Groove sign is seen in-,Syphilis,Dermatomyositis,LGV,SLE,2,single,"Ans. is 'c' i.e., LGV * 'Groove sign of Greenblatt' is pathognomonic of LGV (secondary stage) - when inguinal lymph nodes are enlarged, they are separated by Poupart's ligment, producing a groove.Lymphogranuloma venerum (lymphogranuloma inguinale)* LGV is caused by chlamydia trachomatis, serotypes LI, L2, L3. Serotype L2 is the most common cause. The clinical course of LGV consist of following three stages : -i) First stage (Primary LGV): - Self limited, Single, asymptomatic, painless, non bleeding genital ulcer.ii) Secondary stage: - Painful inguinal lymphadenopathy (Remember - Ulcer is painless but lymphadenopathy is tender & painful). Swollen lymph nodes coalesce to form bubos, i.e., matted lymph nodes. Buboes may rupture to form discharging sinus.# Groove's sign - Enlarge lymph nodes both above and below inguinal ligament.iii) Tertiary LGV (genitorectal syndrome): - Characterized by proctocolitis.Complications of LGV* Esthiomene - Enlargement, thickening and fibrosis of labia.* Elephantiasis of the genitals* Rectal stricture* Systemic - Arthritis, pneumonitis, Perihepatitis",Skin,S.T.D. +5a2c6025-f375-422b-b787-a56ec6b23bca,Hyperchloremic acidosis is seen in-,A,Diarrhoea,DKA,Dehydration,0,single,"The combination of hyperkalemia and hyperchloremic metabolic acidosis is often present, even at earlier stages of CKD (stages 1-3).In patients with diabetic nephropathy or in those with predominant tubulointerstitial disease or obstructive uropathy; this is a non-anion-gap metabolic acidosis.",Medicine,Fluid and electrolytes +8f10721c-3ef4-4c3a-a6eb-77b8da9a9cf0,A patient with cervix cancer is missed by a screening test and later diagnosed with advanced disease. This time interval is called ?,Lead time,Screening time,Serial interval,Generation time,0,multi,Ans. is 'a' i.e. Lead time,Social & Preventive Medicine, +13807e41-a89f-4da3-ab6b-7a64ec0fdb2e,The internal pudendal aery arises from which of the following aeries?,External iliac aery,Inferior vesical aery,Internal iliac aery,Inferior epigastric aery,2,single,The internal iliac aery divides into an anterior and a posterior trunk near the greater sciatic foramen.Branches of ANTERIOR TRUNKUmbilical aeryObturator aeryUterine aeryInferior vesical and vaginal aery (females)Middle rectal aeryInternal pudendal aeryInferior gluteal aeryBranches of POSTERIOR TRUNKIliolumbar aeryLateral sacral aerySuperior gluteal aery,Anatomy, +502328ba-6339-4d52-a737-4f7b57b333b5,Increased Ig A levels are seen in,Alcoholic hepatitis,Alcoholic cirrhosis,Microvescicular fatty change,Macrovesicular fatty change,1,single,"Serum IgA increased in - Alcoholic cirrhosis Serum IgM increased in -Primary biliary cirrhosis (Ref: Clin Exp Immunol.2009 Oct; 158(1): 115-124, Liver.1984 Jun;4(3): 214-8)",Pathology,G.I.T +7217ed20-27bf-4bf1-84d2-6ddb18022234,Rituximab is a monoclonal antibody used for treatment of:-,Non-Hodgkin's lymphoma,Gastrointestinal Stromal Tumors,Chronic Myeloid Leukemia,Acute Myeloid Leukemia,0,single,Rituximab is a monoclonal antibody against CD20. It is used for treatment of: Non-Hodgkin lymphoma Chronic lymphoid leukemia Rheumatoid ahritis SLE Auto-immune hemolytic anemia Idiopathic thrombocytopenic purpura,Pharmacology,Targeted Anticancer Drugs and Immunosuppressants +6b60772d-c91f-450a-92a3-4c26c3c8112a,The least common type of intussusceptions is:,Multiple,Colocolic,Ileoileal,Ileoileocolic,0,single,Types of Intussusceptions (in decreasing order) Ileocolic (77%) Ileo-ileo-colic (12%) Ilioileal (5%) Colocolic (2%): MC in adults Multiple (1%) Retrograde (0.2%),Surgery,Intestinal obstruction +33d6780f-f9b8-4528-bbd9-2b6c065ada10,Most definitive diagnosis of sinusitis is by,X- ray PNS,Proof puncture,Sinuscopy,Transillumination test,2,single,"Sinuscopy is a method of Endoscopic sinus observation or surgery using Nose telescope. It involves the evaluation of nasal and sinus passages using the endoscope called Sinuscope. The Sinuscope has a narrow tube with a built-in camera so that the physician can see the internal details of the sinuses. By observing the sinus, it is possible to diagnose the problems and treat properly.",ENT,Nose and paranasal sinuses +2d906c4a-16b9-494d-83d6-d6831dea8413,"An 18 year old male patient presented to the clinic with pain and swelling in the lower jaw. Intraoral examination +revealed localized gingival bleeding in the right posterior region. On palpation, pulsations can be appreciated. Lesions are also appreciated on the frontal bone on a radiograph. Which condition is present in this patient?",Vascular malformation,Cherubism,Eosinophilic granuloma,Brown tumor,0,single,"Vascular malformation (VM) in bone occurs more frequently than the central hemangioma (CH) of bone. Some 35% of VMs occur in bone, whereas CHs of bone are rare. The CH of bone is a benign tumor that rarely occurs in the jaws; it occurs more frequently in the skull and vertebrae. It may be congenital or traumatic in origin and may be difficult to differentiate from VM. The usual complaint of a patient with a VM or CH is of a slow-growing asymmetry of the jaw or localized gingival bleeding. Numbness and tenderness or pain may also be described. This solitary tumor is found approximately twice as often in female patients, and about 65% occur in the mandible. Some tumors demonstrate pulsation and bruits. Paresthesia is occasionally a feature.",Radiology, +224684fc-43a1-4ea2-b3c4-2137a5b092a9,Most common anterior mediastinal tumor?,Thymoma,Neurofibroma,Pericardial cyst,Bronchogenic cyst,0,single,"Ref: Bailey and Love, 26th edition, P 868Most common anterior mediastinum tumor is thymoma. Choice B is seen in posterior mediastinum. Choice C and D are found in middle mediastinum.",Pathology,Misc. +a11d6f28-7147-4a50-9745-41499c36a2b8,Functional unit of Liver is,Hepatocytes,Poal Tracts,Liver Acinus,Hepatic Lobule,2,single,Anatomical unit of liver - Hepatic Lobule Functional unit of liver- Liver Acinus,Anatomy,Systemic histology +d6ed9456-f50e-4add-af7e-74ff49545af6,Grievous hurt is defined under:,320IPC,318 IPC,321 IPC,375 IPC,0,single,"Ans: a (320 IPC) Ref: Reddy, 21st ed, p. 244Grievous hurt is defined under IPC 320Some important IPC'sIPC 375 - definition of rapeIPC 376 - punishment of rapeIPC 320 - definition of grievous heartIPC 84 - deals with the criminal responsibility of insaneIPC304A - deals with criminal negligenceIPC 304B - dowry deathIPC 300 - defines murderIPC 299 - defines culpable homicideIPC 302 - punishment of murderIPC 304 - punishment of culpable homicide",Forensic Medicine,Introduction to Forensic Medicine and Medical Jurisprudence +6e90b215-167a-448f-89c5-4d05d26c9000,Glaucoma is caused by -,Raised intraocular pressure,Raised intracranial Pressure,Intraocular infection,Orbital mass,0,single,"Ans. is 'a' i.e., Raised intraocular pressure The most common risk factor known is increased intraocular pressure (lOP)o Glaucoma is a group of eye diseases causing optic nerve damage i.e., glaucoma is a chronic, progressive optic neuropathy caused by a group of ocular conditions which lead to damage of optic nerveo Optic neuropathy in glaucoma results in a characteristic appearance of optic disc and a specific pattern of irreversible visual field defects, called glaucomatous changes. It is worth noting that raised IOP without optic neuropathy is not referred to as glaucoma, it is simply called ocular hypertension.So, glaucoma is a type of optic neuropathy, which is usually caused by increased IOP.",Ophthalmology,"General Considerations - Definition, Classification, Pathogenesis" +b93b305f-5080-4b89-b598-f20947050b76,Best indicator for nutritional status for a child is -,Mid arm circumference,Head circumference,Rate of increase of height and weight,Chest circumference,2,single,"Rate of increase of height and weight is the best measure for nutritional status of a child. REF. PARK'S TEXTBOOK OF PREVENTIVE AND SOCIAL MEDICAL 21ST EDITION. 500,501",Social & Preventive Medicine,"obstetrics,pediatrics and geriatrics" +c4a71346-9a2b-4446-bb03-427398f957b8,Aerosolized ribavirin is used in the treatment of bronchiolitis with –,RSV,H.influenza,Pneumococcus,Streptococcus,0,single,,Pediatrics, +5490fe0e-173c-482e-805b-0482ba08204d,Amplification of DNA uses the polymerase chain reaction (PCR) technique. Cation used in PCR is:,Calcium,Lithium,Magnesium,Sodium,2,single,"PCR is an in vitro method for the polymerase-directed amplification of specific DNA sequences using two oligonucleotide primers that hybridize to opposite strands and flank the region of interest in the target DNA. The specificity and yield in amplifying a paicular DNA fragment by PCR reaction is affected by the proper setting of the reaction parameters (e.g., enzyme, primer, and Mg2+ concentration, as well as the temperature cycling profile). Ref: Feng X., Lin X., Brunicardi F.C. (2010). Chapter 15. Molecular and Genomic Surgery. In F.C. Brunicardi, D.K. Andersen, T.R. Billiar, D.L. Dunn, J.G. Hunter, J.B. Matthews, R.E. Pollock (Eds), Schwaz's Principles of Surgery, 9e.",Biochemistry, +5aaf7feb-8094-4590-9a67-2345167bf66c,Which of the following statements about mycosis fungoides is not true?,It is the most common skin lymphoma,Pautriers micro abscesses are common,It has a indolent course and good prognosis,It presents with diffuse erythroderma.,2,multi,C i.e. It has indolent course and good prognosis,Skin, +9a778c55-b44d-46f9-a387-07f4764a5ed4,Babesiosis is transmitted by -,Tick,Mites,Flea,Mosquito,0,single,"Ans. is 'a' i.e., Tick Babesiao Babesiosis is a protozoan disease caused by two species of Babesia : Babesia microti and Babesia divergens.o It is transmitted by loxdid tick.o Babesia infects the RBCs and resides inside the RBCs ( intraerythrocytic). Intraery throcytic infection of Babesiosis is characterised by maltese cross. Maltese cross is a characteristic arrangement of parasites within the erythrocytes - Parasites within erythrocytes are arranged such that pointed ends of four parasites come in contact thereby giving a tetrad configuration resembling a maltese cross. Tetrad forms or 'Maltese cross' appearance is considered pathognomic of Babesiosis.o Clinically Babesiosis presents with chills, fever, mild hepatosplenomegaly, and mild hemolytic anemia,o Treatment includes Atovaquone plus azithromycin or quinine plus clindamycin.o Babesiosis can easily be confused with P. falciparum malaria.Following two features distinguish Babesiosis from malariaPresence of maltese cross in Babesiosis (absent in malaria)Absence of pigment Hemozoin in Babesiosis (present in malaria)Note - Maltese cross is also seen in cryptococcus and aspergillus.",Microbiology,Parasitology +cf77282d-7a63-458f-8dfb-a471cd7534ed,"Manifestation of Acute Dissection include all of the following, Except:",Pericardial effusion,AR,MR,AMI,2,multi,"Answer is C (MR) Mitral Regurgitation is not a feature of Aoic dissection Aoic Regurgitation and Pericardial Tamponade may be seen in proximal dissection Hemopericardium and cardiac tamponade may complicate a dissection involving the ascending aoa (type A lesion with retrograde dissection): Acute Aoic Regurgitation (AR) is an impoant and common complication of proximal dissection - Acute Myocardial Ischemia may be associated with Aoic dissection 'Bowel Ischemia, hematuria and myocardial Ischemia have all benn observed with Aoic dissection'",Medicine, +60506fc9-20d7-414b-a85f-de78719c10fa,Pancytopenia is most common after:,Hepatitis,Infective carditis,Pyelonephritis,Meningitis,0,single,"Hepatitis is the most common preceding infection, and post hepatitis marrow failure accounts for about 5% of etiologies in most series. Patients are usually young men who have recovered from a bout of liver inflammation 1 to 2 months earlier; the subsequent pancytopenia is very severe. The hepatitis is seronegative (non-A, non-B, non-C, non-G) and possibly due to a novel, as yet undiscovered, virus. Ref: Harrison's principle of internal medicine 17th edition, chapter 102.",Medicine, +99f5d0ae-9123-44a6-a1c1-7eda87aba41b,Which is NOT a third generation Cephalosporin,Ceftriaxone,Cefotaxime,Ceftizoxime,Cefuroxime,3,single,"Ans. is 'd' i.e., Cefuroxime Third generation cephalosporins include Parenteral OralCefotaxime - CefiximeCeftizoxime - Cefpodoxime proxetilCeftriaxone - CefdinirCeftazidime - Ceftibuten Cefoperazone Also knowFourth generation cephalosporinsCefepimeCefaparole",Pharmacology,Anti Microbial +19514f92-0244-4363-9913-03e861548b72,Rigor moritis devolops ..... after death :,1/2-1 hrs,1-2 hrs,3-6 hrs,12 hrs,1,single,B i.e. 1 - 2 hours,Forensic Medicine, +c7bc24f2-6ab4-4913-9437-10c2ccbcbb0b,All of the following murmurs may be heard in patients with aoic regurgitation except:,High-pitched decrescendo diastolic murmur.,"Soft, low pitched mid distolic rumbling murmur.",Mid-systolic ejection flow murmur,Pansystolic murmur,3,multi,Answer is D (Pansystolic murmur),Medicine, +12725f6c-3ce1-4ccc-9657-7c7e58a9aa6e,Best investigation to see calcification is:,CT,MRI,X-ray,USG,0,single,"Ans: A (CT ?) Ref: Harrison !v Principles of internal medicine.18th edition.Explanation:CT is the best technique to detect pericardial calcification; however, overpenetrated films, conventional tomography, fluoroscopy, and MRI may be helpful.Plain radiographs have poor sensitivity for detection of coronary calcification and have a reported accuracy as low as 42%High-quality mammography is the best diagnostic tool for the identification of breast calcifications.",Radiology,"Ultrasonography, CT, and MRI" +8fe8bc4d-d411-4281-98fa-f74eaf59212a,Maximum FiO2 which can be given through a nasal oxygen catheter:-,1,0.6,0.2,0.44,3,single,"A nasal cannula is generally used wherever small amounts of supplemental oxygen are required, without rigid control of respiration, such as in oxygen therapy. Most cannulae can only provide oxygen at low flow rates--up to 5 litres per minute (L/min)--delivering an oxygen concentration of 28-44%. Rates above 5 L/min can result in discomfo to the patient, drying of the nasal passages, and possibly nose bleeds (epistaxis). Also with flow rates above 6 L/min, the laminar flow becomes turbulent and the oxygen therapy being delivered is only as effective as delivering 5-6 L/min. The nasal cannula is often used in elderly patients or patients who can benefit from oxygen therapy but do not require it to self respirate. These patients do not need oxygen to the degree of wearing a non-rebreather mask. It is especially useful in those patients where vasoconstriction could negatively impact their condition, such as those suffering from strokes.",Anaesthesia,Anesthesia Circuit +a416ccba-a1a3-4c20-8456-e93759bd8b61,Osteogenesis imperfecta,Is a sex-linked disorder of bones that develop in cartilage,manifests with blue sclera which are pathognomonic of this disease,May be associated with deafness,Has associations with amelogenesis imperfecta,1,single,,Pathology, +8cd5014f-869e-4dc2-ad05-401b60b7ad26,True about neuropraxia,Prolongatinon of conduction velocity,Good prognosis,Both,None,2,multi,"Neuropraxia is due to compression of the nerve,the nerve conduction velocity is increased. It has good prognosis Ref:Guyton and Hall textbook of medical physiology 12th edition,page number:74,75,76",Anatomy,General anatomy +7b3a5354-653e-4558-b990-061ced67975b,Agonistic action at which of the following adrenergic receptor result in the reduction of excess secretion,Beta 1 receptor,Beta 2 receptor,M2 receptor,Alpha 2 receptor,3,single,Ref-KDT 6/e p146 Stimulation of Alpha 2 receptors located on ciliary epithelium reduces secretion of aqueous humor.,Anatomy,Other topics and Adverse effects +2e01c33d-078f-406c-b2a9-7d91a0c7e179,All of the following features are seen in the viral pneumonia except:,Presence of interstitial inflammation,Predominance of alveolar exudate,Bronchiolitis,Multinucleate giant cells in the bronchiolar wall,1,multi,Typical Pneumonia Atypical pneumonia -Bacterial etiology -Alveolar exudate + neutrophilic infilitration -C/F - High grade fever productive Cough Pleuritis Signs of consolidation -Non - bacterial etiology (e.g. Virus) -Interstitial tissue inflammation -C/F- Low grade fever Dry cough Malaise no signs of consolidation,Pathology,Infectious Lung Disease: Pneumonia +d586e17c-d764-4a7b-8d14-d3bc8bdc1c21,Which of the following causes BOTH superior and inferior rib-notching in a chest radiograph?,NF-1,Blalock Taussig shunt,SVC occlusion,Coarctation of aoa,0,multi,Superior rib notching -Polio -Restrictive lung disease -Neurofibromatosis -Connective tissue disease -Osteogenesis imperfecta Inferior rib notching -Thrombosis of aoa -Coarctation of aoa -Blalock Taussig shunt -Occlusion of subclan aery -Pulmonary AV malformation Both Superior and inferior rib notching -NF-1 -Hyperparathyroidism,Radiology,Radiology Q Bank +977cfd21-baf1-4a64-a125-378d92fbdafb,The most accurate diagnostic test for Zollinger-Ellison syndrome (ZES) is,Fasting serum gastrin,Computed tomography (CT) scan,Endoscopy,Secretin stimulation test,3,single,"-All patients with gastrinoma have an elevated gastrin level, and hypergastrinemia in the presence of elevated basal acid output (BAO) strongly suggests gastrinoma. -Patients with gastrinoma usually have a BAO >15 mEq/h or >5 mEq/h if they have had a previous procedure for peptic ulcer. -Acid secretory medications should be held for several days before gastrin measurement, because acid suppression may falsely elevate gastrin levels. 1). The diagnosis of Zollinger-Ellison syndrome (ZES) is confirmed by the secretin stimulation test. An intravenous (IV) bolus of secretin (2 U/kg) is given and gastrin levels are checked before and after injection. - An increase in serum gastrin of 200 pg/mL or greater suggests the presence of gastrinoma. Patients with gastrinoma should have serum calcium and parathyroid hormone levels determined to rule out multiple endocrine neoplasia type 1 (MEN1) and, if present, parathyroidectomy should be considered before resection of gastrinoma.",Surgery,Stomach & Duodenum +10957cbd-4a0e-4d84-afef-f08ee2a1cf1a,What is the average number of fleas of each species per rodent called -,General Flea index,Specific Flea index,Incidence of Fle species,Flea infestation Rate,1,multi,"

It is the average number of each species per rat/rodent. Reference:Park&;s textbook of preventive and social medicine,K.Park,23rd edition,page no:294. <\p>",Social & Preventive Medicine,Communicable diseases +6c68e78f-09e1-4dc6-8a22-b2469804b24e,Periosteal reaction in a case of acute osteomyelitis can be seen earliest at: March 2012,5 days,10 days,15 days,20 days,1,single,Ans: B i.e. 10 days The earliest sign to appear on the X-ray is a periosteal new bone deposition (periosteal reaction) at the metaphysis. It takes about 7-10 days to appear Osteomyelitis Earliest site of involvement: Metaphysis Diagnosis of acute OM: Blood culture (positive in 65% cases) Periosteal reaction seen in acute OM appears after: 10 days MC organism in acute OM: Staph. aureus Brodies abscess: - Equilibrium between host & organism; - Abscess cavity remains without fuher enlargement,Surgery, +65aaa955-47b2-4791-b9c2-e3272ba7b4e1,Which of the following is not associated with zinc deficiency: March 2005,Delayed wound healing,Loss of libido,Pigmentation,Sexual infantilism,2,single,"Ans. C: Pigmentation Acrodermatitis enteropathica is a rare inherited form of zinc deficiency, characterized by periorificial and acral dermatitis, alopecia, and diarrhea. Symptoms of Zinc Deficiency Poor Immune system Weight loss Intercurrent infections Hypogonadism in males Lack of sexual development in females Growth retardation Dwarfism Delayed pubey in adolescents Rough skin Poor appetite Mental lethargy Delayed wound healing Sho stature Diarrhea One easily recognized sign which may be caused by zinc deficiency is white spots, bands, or lines on fingernails (leukonychia). Some women may have multiple parallel white bands or lines on the fingernails marking menstrual cycles when marginal zinc deficiency was present.",Medicine, +abf516bd-aeec-4fee-a74e-4fcaf83bdeec,Preimplantataion genetic diagnosis is used for,Fetal gender determination,Single gene mutation analysis,HLA typing,Karyotyping,1,single,"Pre implantation genetic diagnosis is used for daignosis of single gene disorders such as cystic fibrosis, β - thalassemia and hemophilia.",Gynaecology & Obstetrics, +0a1b9d20-8102-4e64-a537-5cc6068abea5,Which of the following about phenytoin is true ?,It follows zero order kinetics,Not an hepatic enzyme inducer,It is excreted unchanged in urine,It is not teratogenic,0,multi,"Ans. is 'a' i.e., It follows zero order kinetics Phenytoin is metabolized in liver by hydroxylation and glucuranide conjugation (option 'c' is incorrect) o This drug follows saturation kinetics (kinetics changes from first order to zero order within therapeutic concentrations) Phenytoin is an enzyme inducer and induces the metabolism of many drugs. Phenytoin is a known teratogenic and produces fetal hydantoin syndrome.",Pharmacology, +93e16540-7e0c-4dd7-aa21-caeb07a1d676,False about stagardt's disease,Autosomal dominant,No family history,Beaten bronze appearance fundus,ERG and EOG both are normal,0,multi,Stagardt's disease is autosomal recessive.,Ophthalmology, +c554afb1-9705-4a40-b97d-30e243390fc3,"All of the following are killed vaccines, EXCEPT:",Salk,Hepatitis B vaccine,17-D Vaccine,HDCV,2,multi,"17-D vaccine is a live attenuated vaccine used to control yellow fever. Killed vaccines are used to control typhoid, cholera, peusis, plague, rabies, salk (polio), influenza, hepatitis A, B, japanese encephaltis and KFD. Ref: Park's Textbook Of Preventive and Social Medicine 20th edition page 98.",Social & Preventive Medicine, +81697ce5-1551-43f9-a2c6-b41cf41fb01b,Kerley B line are seen at,Apex,Cardiophrenic angle,Lung fissure,Pleural surface,3,single,"Ans. is 'd' i.e., Pleural surface Kerley's line Kerley's line is a linear opacity which, depending on its locaton, extent and orientation, may be be fuher classified as :? Kerley's A lines o These are thin, non-branching lines radiating from hilum. These lines are 2-6 cm in length. These lines are found in the mid and upper zones of the lung fields pleural. These lines are due to the thick interlobar septa. Kerley's B lines (septal lines) These are transverse (Horizontal) lines at the lung base pleural. These are 1-2 cm in length and are perpendicular to pleura. Kerley's B line is due to the thickening of interlobar septa due to edema caused by pulmonary venous hypeension (congestion), as occurs in left ventricular failure or mitral stenosis",Medicine, +397b415a-2a93-42de-a74c-727db8f8ea83,All of the following are true about BCG vaccine except.,WHO recommends Danish 1331 strain of M.bovis,Given intradermally,Normal saline is a diluent,Site of injection should be cleaned with spirit,3,multi,Site of injection should be cleaned with soap but not with disinfectant or spirit.,Microbiology, +202f3a45-d0cd-490a-92f8-7dcf2bf230b3,Commonest etiology of erythema multiforme is -,Viral,Bacterial,Food,Drugs,0,single,A. i.e. Viral,Skin, +79e04d67-af42-4b0e-b242-cd83473eed50,Apex national institute of ophthalmology is,Advance eye care PGI Chandigarh,RIO,"Dr. R P Centre for Ophthalmic Sciences , Delhi","Shankara Netralaya , Chennai",2,single,"Dr RP Centre for Ophthalmic Sciences, New Delhi has been designated as the apex national institute of ophthalmology.",Ophthalmology,Community Ophthalmology +5b6dc8b7-df2e-4cee-8466-bdafd94dc6c6,Which laser is used in the management of after cataracts:,Argon,Krypton,Nd-YAG,Excimer,2,single,Ans. Nd-YAG,Ophthalmology, +ae109a16-29aa-4d8f-89e7-24dc53863115,Which of the following statements about Kaposi sarcoma is not true?,The most common site is scalp,Lesions are dark blue or purple mostly,They are usually multifocal,Lymph nodes may be involved,0,multi,"Ans. A. The most common site is scalpKaposi sarcoma is mostly seen in extremities: most commonly on feet and occasionally on hands, ears, and nose.* Lesions are usually dark blue or purple and they may blanchewhen tumid (swollen).* Locally aggressive lesions can ulcerate, fungate or leave pigmented scars.* Lesions are usually multifocal which eventually fuse to form plaques and tumors of several centimeters in size.* Lymph nodes, mucosae and viscera may be involved as the disease progresses, although this can occur without skin involvement.Characteristic purple-colored plaques on the alar and tip of the nose in an HIV-positive female patient.",Skin,Miscellaneous +b99f828c-44db-4bb5-b862-ead7b0034e98,All of the following veins lack valves except,Femoral vein,Poal vein,IVC,Dural venous sinuses,0,multi,"Veins which do not have valves are:IVCSVCHepatic, ovarian, uterine, renal, emissary, cerebral, pulmonary, and umbilical veinsPoal venous system is a valveless system",Anatomy, +1da9268f-ba06-4576-8f19-ca634a3e0dce,The best laboratory test to use in the diagnosis of Lupus vulgaris in the oral cavity is:,Bacterial smear,Blood studies,Biopsy,Blood chemistry,2,single,,Pathology, +7b7870d5-cd64-4e63-b9d5-cdf8aa81f9c5,Risk factor for melanoma are all of the following EXCEPT: March 2013,Exposure to UV radiation,Black people,First degree relative with melanoma,Multiple or dysplastic naevi,1,multi,Ans. B i.e. Black people Melanoma is commoner in fair complexion Malignant melanoma/ MM Mode of spread of MM: Lymphatic channels or hematogenous Cutaneous melanoma arises from: Epidermal melanocytes MC site involved in lentigo maligna (least common): Face MC type of MM: Superficial spreading Most malignant type of MM: Noduar melanoma,Surgery, +c939bbeb-af27-43ec-8ad1-d7f70f5543eb,Preferred fluoroquinolone against m. leprae is -,Ofloxacin,Pefloxacin,Ciprofloxacin,Moxifloxacin,0,single,"Ans. is 'a' i.e., Ofloxacin Drugs acting on M. leprae* Established agents used to treat leprosy include dapsone (50-100 mg/d), clofazimine (50-100 mg/d, 100 mg three times weekly, or 300 mg monthly), and rifampin (600 mg daily or monthly. Of these drugs, only rifampin is bactericidal.* The sulfones (folate antagonists), the foremost of which is dapsone, were the first antimicrobial agents found to be effective for the treatment of leprosy and are still the mainstay of therapy.* Other antimicrobial agents active against M. leprae in animal models and at the usual daily doses used in clinical trials include ethionamide/prothionamide; the aminoglycosides streptomycin, kanamycin, and amikacin (but not gentamicin or tobramycin); minocycline; clarithromycin; and several fluoroquinolones, particularly preferred is ofloxacin.* Next to rifampin, minocycline, clarithromycin, and ofloxacin appear to be most bactericidal for M. leprae, but these drugs have not been used extensively in leprosy control programs. Most recently, rifapentine and moxifloxacin have been found to be especially potent against M. leprae in mice. In a clinical trial in lepromatous leprosy, moxifloxacin was profoundly bactericidal, matched in potency only by rifampin.",Pharmacology,Anti Microbial +7d6c6949-7c51-44f2-9c75-3be538acae26,When the patient fails to understand normal speech but can understand shouted or amplified speech the hearing loss is termed as,Mild hearing loss,Moderate hearing loss,Severe hearing loss,Profound hearing loss,2,single,"DEGREES OF HEARING LOSS 0-25dB-Not significant impairment- no significant difficulty with faint speech 26-40dB-Mild impairment-Difficult with faint speech 41-55dB-Moderate impairment -Frequently difficult with normal speech 56-70dB-Moderately severe impairment-Frequently difficult with loud sound 71-91dB-Severe impairment-Can understand only shouted or amplified sound Above 91-Profound impairment-Usually cannot understand even amplified sound Ref: Diseases of EAR, NOSE AND THROAT by PL DHINGRA - 6th Edition. Page no. 39",ENT,Ear +aa99ff73-6f36-4c45-a44a-86a3bbf319fb,Local anaesthetics :,Block the release of neurotransmitters,Block the influx of sodium into the cell,Increase the release of inhibitory neurotransmitters,Inhibit the efflux of sodium from neurons,1,single,,Pharmacology, +cac8d3b2-1ef7-459d-8ee5-7582fb4d7f0e,Gingival sulcus is V shaped and it barely permits the entrance of a,Periodontal hoe,Periodontal curette,Periodontal probe,Periodontal sickle,2,single,,Dental, +70e492ea-3cbe-4dbb-a180-fc3bdd1df593,"A child with three days history of upper respiratory tract infection presents with stridor, which decreases on lying down postion. What is the most probable diagnosis -",Acute Epiglottitis,Laryngotracheobronchitis,Foreign body aspiration,Retropharyngeal abscess,1,single,"Ans. is 'b' i.e., Laryngotracheobronchitis This child has : - i) 3 days history of upper respiratory tract infection. ii) Followed by stridor These features suggest the diagnosis of croup. Clinical manifestations of Croup Most patients have an upper respiratory tract infection with some combination of - Rhinorrhea Pharyngitis Mild cough Low grade fever o After 1-3 days signs and symptoms of upper respiratory tract obstruction become apparent - Barking cough Hoarseness Inspiratory stridor",Pediatrics, +ccfdaf28-aa1a-49c4-84b1-943e68d6b648,Predominant bacteria found in two days old plaque,Streptococci,Bacteroides,Spirocheates,Actinomyces,0,single,,Dental, +d553ada9-cbb7-4fab-95f3-c4cc4ce536dd,Most common movemnet force involved in fractures of the spine is which of the following?,Flexion,Extension,C Rotation,Compression,0,single,"Flexion injury is the commonest spinal injury. Ref: Essential ohopedics by Maheshwari 3rd Edition, Page 144,259,260.",Surgery, +dadacc3d-d999-45e2-a343-f58debb1da3b,Which arm of tRNA binds it to vibosomal surface,DHU arm,Pseudouridine arm,Acceptor arm,Anticodon arm,1,single,Pseudoridine arm of tRNA is involved in binding of aminoacyl tRNA to ribosomal surface.,Biochemistry, +e209480e-ce98-4f4e-9bd6-883ee8cc4bfa,Benefits of LNG lUCD are all except :,Management of menorrhagia,Contraceptive effect,Hormone replacement therapy after menopause,stage 2 endometrial cancer,3,multi,"Mirena now used as first line of treatment of menorrhagia, in addition to the contraceptive effect. Another use is in providing progestogens component of hormone replacement therapy after menopause. THE TEXTBOOK OF GYNAECOLOGY SHEILA BALAKRISHNAN SECOND EDITION PAGE NO 379",Gynaecology & Obstetrics,Contraception +53c85cf9-4ec1-4485-91cf-1bd3b077dbaf,Most common drug induced Extra Pyramidal Syndrome includes,Dystonia,Chorea,Tardive dyskinesia,Hemiballismus,0,multi,"(A) Dystonia # Drug Induced & Tardive Movement Disorders> Movement disorders secondary to pharmacological agents represent a large number of extrapyramidal disorders seen by neurologists and psychiatrists in the outpatient setting.> Involuntary movements, including tremor, chorea, athetosis, dyskinesias, dystonia, myoclonus, tics, ballismus and akathisia, may be symptoms of primary neurologic disease or occur secondary to pharmacotherapy CHARACTERISTICS* TremorRhythmic. Oscillatory movement categorized according to its relationship to activity or posture* ChoreaIrregular, unpredictable brief jerky movements* AthetosisSlow, writhing movements of distal parts of limbs* DyskinesiasRecessive abnormal involuntary movements* DystoniaSlow sustained, posturing or contractions of a muscle or group of muscles* MyoclonusRapid, brief shock like muscle jerks* TicRepetitive, irregular stereotype movements or vocalizations* BallismusWild flinging or throwing movements* AkathisiaSubjective sensation of restlessness often associated with inability to keep still. Easily confused with psychiatric symptoms such as agitation, hyperactivity and anxiety> Central stimulants that act as indirect dopamine agonists such as amphetamine> Levodopa, a precursor of dopamine> Direct dopamine agonists such as bromocriptine> Presynaptic dopamine antagonists (dopamine depleting agents) such as reserpine> Neuroleptics such as haloperidol (Haldol) or chlorpromazine (Thorazine), and other medications such as metoclopramide (Reglan) which antagonize or block central dopamine receptors> By far, the most common cause of drug-induced and tardive syndromes are those that block or antagonize dopamine receptors, usually the neuroleptics.",Medicine,Miscellaneous +a4294bee-22af-4c61-bdc0-be1a892d1e35,The earliest manifestation of increased intracranial pressure following head injury is:,Ipsilateral pupillary dilatation,Contralateral pupillary dilatation,Altered mental status,Hemiparesis,2,single,"Early signs of elevated iCP include drowsiness and a diminished level of consciousness (altered mental status). Coma and unilateral papillary changes are late signs and require immediate intervention. Ref: Harrison's Internal Medicine, 16th Edition, Page 1633; Clinical Hepatology: Principles and Practice of Hepatobiliary Diseases By Henryk Dancygier, Scott L. (FRW) Friedman, H. D. (CON) Allescher, U. (CON) Beuers, Volume 2, 2010, Page 938",Surgery, +9068aea8-3633-4c43-b958-41861be80599,Function of external oblique muscle:,Anterior flexion of veebral column,Active expiration,Closure of inguinal ring,All of the above,3,multi,"External oblique: Most superficial muscle which originates from the outer pa of the 5th to 12th ribs on each side of the rib cage. This muscle then runs diagonally down each side and connects to the iliac crest, linea alba, and the pubis. Function: External oblique contributes to the maintenance of abdominal tone, increasing intra-abdominal pressure (as in active expiration), and lateral flexion of the trunk against resistance. Bilateral contraction flexes the trunk forward and aid in movement of spine and back. Contraction of external oblique muscle approximates the two crura (medial and lateral) of superficial inguinal ring like a slit valve to maintain the integrity of inguinal canal.",Anatomy,"Abdominal wall ,Inguinal and Femoral region" +178a2e17-db01-44b3-9c8a-b5e62b5d1e8f,P wave is absent in-,Atrial fibrillation,Atrial asystole,Ventricular fibrillation,Ventricular tachycardia,0,single,"Absence of the P wave with a flat baseline may indicate: Fine atrial fibrillation Sinoatrial arrest (with a secondary escape rhythm ) in ventricular fibrilation ,ventriculat tachycardia and atrial asystole,a waves are present Ref Harrison 20th edition pg 1423",Medicine,C.V.S +757489e2-1d81-48b0-b83c-9015ebb54943,A Poly A base sequence would be most likely found at the,5' end of a prokaryotic messenger RNA (mRNA),3' end of a prokaryotic mRNA,5' end of a eukaryotic mRNA,3' end of a eukaryotic mRNA,3,single,The addition of a poly A tail to the 3' end is one of the post-transcriptional modifications that occurs in the processing of eukaryotic messenger RNA (mRNA). A cap consisting of a guanosine derivative is attached to the 5' end. Intervening sequences (introns) are removed by splicing. All of these processing events occur in the nucleus of eukaryotes. Prokaryotic mRNA undergoes none of these modifications.,Surgery, +0297e7f0-4705-4b8d-9289-98ec58229859,Type I RPGN is seen in,Cryoglobulinemia,SLE,Goodpasture's syndrome,Wegner's granulomatosis,2,single,Type I RPGN is anti - GBM mediated and it is seen in Goodpasture's syndrome.,Pathology, +a8f0ecfa-295c-46ae-9357-5aa92429e19d,Bradycardia is seen with:,Midazolam,Epinephrine,Succinyl choline,Dopamine,2,single,"Ans: C (Succinyl choline) Ref: Ajay Yadav, Short textbook of Pediatrics, 1st editionExplanation:Cardiac Sideeffects of Anesthetic DrugsDrugsCardiac Side effectsMidazolamMinimal reduction in Heart rate, Blood pressure and cardiac outputEpinephrineProduces tachycardia, hypertension and ventricular arrhythmiasSuccinyiCholineIt produces muscarinic effects, similar to Acetyl cholineIt causes Profound BRADYCARDIA, so atropine should be given prior to use of Succinylcholine Choline, especially in childrenDopamineProduces tachycardia, hypertension and ventricular arrhythmiasThiopentoneit causes Hypotension, which is more because of venodilatation and direct depression of vasomotor centreDirect myocardial depressant",Unknown, +3cd896db-2ff3-451c-9e7e-e6865e50453d,Infidelity & jealousy involving spouse is the thought content of which disorder -,Capgras syndrome,Othello syndrome,Hypochondrial paranomia,Declerambault's syndrome,1,single,,Psychiatry, +eed4f2ca-a443-4de6-8bba-51cf3098269d,Failure of migration of neural crest cells is seen in:,Albinism,Congenital megacolon,Odontomes,Adrenal tumour,1,single,"Ans: b (Congenital megacolon)Ref: Bailey & Love, 24th ed, p.l 153 & 23rd ed, p. 1027",Surgery,Small & Large Intestine +e07a117b-53b8-4029-8b7e-4d97bd1ef590,The signs of malignant transformation in osteochondroma are all except,Pain,Weight loss,Increase in size,Increase in thickness of cailage cap,1,multi,Ans. b. Weight loss,Surgery, +80a59636-4395-44dd-b040-48c3ae1dfb49,The number 12 in a 3 unit formula 12-6-8 indicates the blade is,12 mm in length,1.2 mm in length,12 mm in width,1.2 mm in width,3,single,,Dental, +ff3bc438-722d-4724-8f52-5f7597ea4bd3,Prophylaxis for health personnel working in a plague ward is -a) Vaccineb) Tetracycline throughout the dutyc) A cource of tetracyclined) Vaccine and Erythromycine) Observation,ac,a,ab,bc,2,single,,Social & Preventive Medicine, +58a3c43f-c28a-4679-96ad-f95dafd52835,Drugs known to cause Steven Johnson syndrome include the following except:,Cephalosporins,Ibuprofen,Carbamazepine,Paracetamol,3,multi,"Drugs that most commonly cause SJS/TEN Antibiotics Antifungals Antivirals Sulfonamides, e.g., cotrimoxizole; beta-lactams i.e. penicillins, cephalosporins Imidazole antifungals Nevirapine (non-nucleoside reverse-transcriptase inhibitor) Allopurinol Nonsteroidal anti-inflammatory drugs (NSAID) (oxicam type mainly) Naproxen Ibuprofen Anti-convulsants Carbamazepine Phenytoin Phenobarbital Valproic acid Lamotrigine",Dental,Blistering disorders +70ccdb13-7ec5-4de9-aeaf-6ffb4d184372,Best prognosis in nerve injury,Neuropraxia,Axonotemesis,Neurotemesis,Complete transaction,0,single,"Neurapraxia is a disorder of the peripheral nervous system in which there is a temporary loss of motor and sensory function due to blockage of nerve conduction, usually lasting an average of six to eight weeks before full recovery.Ref: Ganong&;s review of medical physiology 23rd edition",Physiology,Nervous system +b6be509b-8060-4678-80d8-605dccfc56a5,Bedside test for mental status-,MMSE,GCS,MMPI,WAIS,0,single,"Ans. is 'a' i.e., MMSE o The most w idely used test for bedside evaluation of the mental status is folstein's mini mental state examination (MMSE).",Psychiatry,Organic Mental Disorder +d73190cf-3f68-4d02-a7f5-0d6fac8cdcdb,A 70-year-old man comes to casualty with urinary retention and back pain. Which investigation should be performed -,Serum acid phosphatase,Serum Calcium,Serum alkaline phosphatase,Serum electrophoresis,0,single,"pt. is suspected to have Ca Prostate as Prostatic Ca is the most common malignant tumor in men over 65 yrs. of age. Symptoms of Bladder outlet obstruction and back pains (due to bony metastasis in the pelvis & lumbar veebra) indicate towards prostate Ca. Serum acid phosphates is a tumor marker of prostate Ca. But now serum acid phosphates assay has been superseded by PSA assay (Prostate specific antigen). Prostate-specific antigen It is a glycoprotein produced only in the prostatic cells (both benign & malignant). It facilitates liquefaction of semen. It is neither sensitive nor specific for early prostate carcinoma (it is prostate specific and not prostate cancer specific), neveheless it gives some help in making a diagnosis. Normal serum level - less than 4 mg/ml 4 - 10 mg/ml - this range is common for both BHP and Ca. More than 10 mg/ml - approx 75% will have cancer. Since PSA is not specific for Ca, PSA Velocity & PSA density is used to detect Prostate cancer. PSA velocity is the rate of change in PSA levels over time and is expressed most commonly as the PSA doubling time. For men with a PSA above 4, PSA velocity of more than .75 mg/ml year is suggestive of Ca. While for those with lower PSA levels, rates above 0.5 mg/ml, per year should be used to advise biopsy. PSA density is calculated by dividing the serum PSA by the estimated prostate weight (measured by TRUS). It was developed to correct for the contribution of BPH to the total PSA level. Values < 0.10 are consistent with BPH. > 0.15 suggest cancer Ref : Bailey & Love 25/e p1356",Anatomy,Urology +a188906b-d986-4bad-9952-126f99fd2a30,Contraindications of circlage operation are all except:,Leaking membranes,Features of amnionitis,History of vaginal bleeding,Pregnancy beyond 14 weeks,3,multi,,Gynaecology & Obstetrics, +2a60f39a-e630-4fe9-9836-6a32f37d3809,Elaboration of inactivating enzymes are the impoant mechanism of drug resistance among all of these antibiotics except,Quinolone,Pencillin,Chloramphenicol,Aminoglycoside,0,multi,Refer KDT 6/e p688 Resistance to fluoroquinolone is mediated by mutation in DNA gyrase,Pharmacology,Chemotherapy +9f1744a7-19f6-4169-8465-c6a816133865,A patient presents with unconjugated hyperbilirubinemia and presence of urobilinogen in urine. Which amongst the following is the least likely diagnosis -,Hemolytic jaundice,Crigler Najjar syndrome,Gilbert's syndrome,Dubin Johnson syndrome,3,single,,Medicine, +a6c666a5-7859-43f7-8ddd-6422068d04e5,Which among the following is TRUE about confounding factor?,It is found equally between study and the control groups,It is itself a risk factor for the disease,Confounding can be eliminated by selecting a small group,It is associated with either the exposure or the disease,1,multi,"A confounding factor is defined as one which is associated both with exposure and disease, and is distributed unequally in study and control groups. More specifically a confounding factor is one that, although associated with exposure under investigation, is itself, independently of any such association, a risk factor for the disease. Ref: Park's Textbook of Preventive and Social Medicine, 19th edition, Page 67.",Social & Preventive Medicine, +8293f730-fd62-4d63-bfd7-c758905013c1,Investigation of choice for esophageal rupture is,Dynamic MRI,Rigid esophagoscopy,Barium contrast swallow,Water soluble low molecular weight contrast swallow,3,multi,"Gastrografin (water soluble) is preferred to prevent extravasation of barium into the mediastinum or pleura. If no leak is seen, a barium study should follow",Surgery,All India exam +b59118d8-34fc-4814-935c-523da1791c48,Dobutamine differs from dopamine in that,It has good blood-brain barrier penetrability,It causes pronounced tachycardia,It does not activate adrenergic b receptors,It does not activate peripheral dopaminergic receptors,3,single,"Dopamine (DA):- It is a dopaminergic (D1 and D2) as well as adrenergic a and b1 (but not b2) agonist. Dobutamine:- A derivative of DA, but not a D1 or D2 receptor agonist. Though it acts on both a and b adrenergic receptors. Ref:- kd tripathi; pg num:-134",Pharmacology,Autonomic nervous system +0ee1e184-ebb7-4f8d-8029-fc644944ce06,Garlicky odour in the gastric contents seen in which non metallic poisoning,Sulphur,Phosphorus,Iodine,Chlorine,1,multi,"Postmoem appearance Esophagus and stomach shows inflammatory changes and luminous material may be seen in the GIT. There may be garlicky odour in the gastric contents. There may be jaundice, and fatty change in the liver. Liver shows centrilobular steatosis and neurosis. If the person survived for one week or so, there may be yellow atrophy of liver and cloudy swellings of kidney. Ref: FORENSIC MEDICINE AND TOXICOLOGY Dr PC IGNATIUS THIRD EDITION PAGE 432",Anatomy,Poisoning +5f4b321f-8d37-411c-b1ab-7262eb4a2a2e,Abnormal function of epithelial chloride channel protein is the cause of,Ehlers Danlos syndrome,Marfan syndrome,Cystic fibrosis,Diabetes insipidus,2,single,,Medicine, +4aba0ab7-8db0-4de7-99be-fdf787ed3f0c,The capsule of cryptococcus neoformans in a CSF sample is best seen by -,Gram stain,India ink preparation,Giemsa stain,Methenamine-silver stain,1,single,"Cryptococcus is the capsulated yeast. +Among the given staining techniques, India ink preparation is the best staining technique used for demonstration of capsule (negative staining) - sensitivity of the technique: 60–75%. + +Other capsular staining techniques are: + +10% Nigrosin staining +Modified India ink preparation with 2% chromium mercury +Alcian blue staining + +Methanamine silver and Periodic acid- Schiff – used for tissue sample. +Sensitivity of various diagnostic tests- Harrison 18/e p1652 + +Cryptococcal antigen detection in CSF—90% +Blood culture: 10—30% in non-HIV patients and 60% in HIV patients +Sputum culture: 10% +Sputum antigen detection: 30%",Microbiology, +aa2c69c6-22ee-43c0-8c4b-6c73a2ff84b0,which is not a radiographic finding of CHF,Kerley B lines,Kerley A lines,Cardiomegaly,Oligemia,3,single,ref : harrisons 21st ed,Radiology,All India exam +0a8d5e9a-0c63-4550-adde-ea69ba439d37,Alternating RBBB with Left anterior hemiblock is seen in,1' degree hea block,Complete hea block,Mobitz type II block,Bi-fascicular block,3,single,"Ans. is 'd' i.e., Bi-fascicular block Bifascicular block - combination of RBBB with either left anterior hemiblock or left posterior hemiblock. Tri fascicular block - RBBB plus either LAHB/LPHB+ first degree AV block. Complete hea block destruction of - AV node leading to AV dissociation",Medicine, +69fa1e5c-3053-49cb-b4f0-2de8d464d5ef,Which of the following is false about cholangiocarcinoma,Investigation of choices MRCP,Staging is done by PET CT,Diagnosis is done by CECT,Choledochal cyst is a risk factor.,1,multi,Diagnosis is made by MRCP,Surgery, +bdcc584c-b77b-461b-84f5-21a04ab33437,All of the following heart sounds occur shortly after S2 except,Opening snap,Pericardial knock,Ejection click,Tumor plop,2,multi,,Medicine, +af87a839-8794-47b6-bc5c-131da1b3f168,Which of the following clinical laboratory observations is suggestive of Hanup disease?,Burnt sugar smell in urine,High plasma phenylalanine levels,Extremely high levels of citrulline in urine,High fecal levels of tryptophan and indole derivatives,3,single,High fecal levels of tryptophan and indole derivatives Hanup disease is autosomal recessive metabolic disorder also known as pellagra like dermatosis. This affect the absorption of nonpolar( neutral ) amino acids specially tryptophan. So there is no absorption of tryptophan.,Physiology,All India exam +09e17233-d63c-4399-bf4e-1350747da141,"All are true about pathogenesis of Ovarian Hyperstimulation Syndrome, EXCEPT?",Due to ovarian enlargement and fragility,Increase in intravascular volume,Role of VEGF and inflammatory cytokines,Secondary to infeility treatment,1,multi,"Symptoms of OHSS are the result of ovarian enlargement and fragility, extravascular fluid accumulation, and intravascular volume depletion (and not increase as stated in option b). Fluid shifts that accompany OHSS are due to increased protein-rich fluid secretion from the stimulated ovaries, increased renin and prorenin within follicular fluid, increased capillary permeability and VEGF",Gynaecology & Obstetrics,Ovarian Hyperstimulation Syndrome (OHSS) +26a1bbfc-a56e-4059-b43a-768cc11e8b32,Biological value of a protein is related to,Nitrogen content,amino acid content,Sulphur content,Energy content,0,single,Ans) a (Nitrogen content) Ref paik 20th ed p 549Biological value of a protein is Retained N2 / Absorbed N2 x 100Protein efficiency ratio = Wt gain in gms / Gram of protein consumedNet protein utilization =Digestibility Coefficent x Biological value100 Aminoacid score =No of mg of one AA/gm of proteinNo of mg of same AA/gm of egg proteinx 100Quality of protein is assessed by comparison to the reference protein which is usually egg protein.,Social & Preventive Medicine,Nutrition and Health +595cf09b-34c6-4d89-9033-6ff3ed0b38d5,The study for correlation of genetic disease to consanguinity -,Case Contral Study,Cohort study,Cross-sectional study,Case report,0,single,"Ans. is 'a' i.e., Case Control Study "" The association of consanguinity with complex disorders can be studied using different approaches. For example, epidemialogial surveys could compare the frequency of a disorder in the progeny offirst cousin parents with that of unrelated parents, whereas case-control studies could compare the rates of first cousins among affected individuals and controls"".",Social & Preventive Medicine,Epidemiological Study +9752d8e3-e260-40dd-97ae-d846f413301e,Von Gierke&;s occurs due to deficiency of,Glucose-6-phosphatase,Liver Phosphorylase,Muscle phosphorylase,Debranching enzyme,0,single,"Glycogen storage disease type I (GSD I) or von Gierke&;s disease, is the most common of the glycogen storage disease. This genetic disease results from deficiency of the enzyme glucose-6-phosphatase and has an incidence in the American population of approximately 1 in 100,000 bihs.Ref: DM Vasudevan, 7th edition, page no: 128",Biochemistry,Metabolism of carbohydrate +805e7f24-2c61-4783-92de-e8757acfc270,"A 38 year old male has paroxysmal hypeension. He is subsequently found to have medullary carcinoma of the thyroid, pheochromocytoma, and mucosal neuromas. Parathyroid involvement is not noted. What is the most likely diagnosis?",MEN type I,MEN type II,MEN type III,Sipple's syndrome,2,single,"MEN III, also known as MEN IIb, is characterized by medullary thyroid carcinoma, pheochromocytoma, and mucosal neuromas.Wermer's syndrome is also called MEN type I . It is characterized by pancreatic (insulinoma), pituitary, and parathyroid involvement.Sipple's syndrome, or MEN type II , is similar to MEN III, but it has parathyroid involvement (tumor or adenoma) as opposed to neuromas. Ref: Wyatt C., Butterwoh IV J.F., Moos P.J., Mackey D.C., Brown T.G. (2008). Chapter 18. Endocrine Pathology. In C. Wyatt, J.F. Butterwoh IV, P.J. Moos, D.C. Mackey, T.G. Brown (Eds), Pathology: The Big Picture.",Pathology, +9a64eaa1-2bac-44e9-9dce-c0d1c64c258a,"Mrs Shikha, 50-years-old woman is diagnosed with cervical cancer. Which lymph node group would be the first involved in metastatic spread of this disease beyond the cervix and uterus? NOT RELATED -anatomy",Common iliac nodes,Parametrial nodes,External iliac node,Paracervical or ureteral nodes,3,single,.,Pharmacology,All India exam +b17a2718-0ecd-4f2a-a95f-782637289638,A viral infection causes damage to both hippocampi in a patient. This damage would cause the patient to exhibit functional deficits in,Recalling an old declarative memory,Recalling an old procedural memory,Forming a new sho-term memory,Forming a new long-term memory,3,multi,"The hippocampus is crucial for the formation of long-term (declarative) memory. Without the hippocampus, sho-term memory is intact but the conversion to long-term does not take place.The retrieval of stored declarative memory does not require the hippocampus. The hippocampus is not needed for the retrieval of procedural memory.Ref: Guyton; 13th edition",Physiology,General physiology +592dd58a-ef4f-4259-b81d-44a5fc989338,Type of collagen maximum in skin:-,Type I,Type II,Type III,Type IV,0,single,"Collagen - the most abundant protein in mammals contains 4-hydroxyproline and 5-hydroxylysine. Vitamin C plays the role of a coenzyme in hydroxylation of proline and lysine while protocollagen is conveed to collagen The hydroxylation reaction is catalysed by lysyl hydroxylase (for lysine) and prolyl hydroxylase (for proline) This reaction is dependent on vitamin C, molecular oxygen and a-ketoglutarate Type Distribution I Noncailaginous connective tissues, including bone, tendon, skin II Cailage, vitreous humor III Extensible connective tissues, including skin, lung, vascular system IV Basement membranes",Biochemistry,NEET 2019 +eec09aec-46e6-477d-bed3-4872b61f97fd,Rotameters,Depend on laminar low for their accuracy,Will only function when upright,Are constant pressure drop-constant orifice devices,Are not accurate below 1L/min,1,single,Rotameters are constant pressure drop-variable orifice devices. They are accurate at values as low as 200mL/min both laminar and turbulent flow determine their accuracy .,Anaesthesia, +2f52d409-c4df-4539-8e4b-2eed20e3c99c,Which of the following is true of Wilson's disease all except -,Autosomal recessive,Serum ceruloplasmin level < 20 mg/d1,Urinary coppor excretion < 100 microgram/c11,Zinc acetate is used as maintence therapy,2,multi,"Urinary copper excretion <100 microgram/di Symptomatic pts. of Wilson disease invariably have urine copper levels > 100 pg per day. Wilson disease is an autosomal recessive disorder caused by mutation in the ATP 7B gene (a copper transpoing ATPase) Diagnosis - The gold standard for diagnosis is Liver biopsy with quantitative copper assayQ. Other diagnostic tests used are ? - Serum ceruloplasmin levelQ - KF rings(2 - Urine copper excretiono - DNA Helpful-ye analysisQ Serum copper values have no diagnostic value, since they may be low, normal or elevated depending upon the stage of evolution of disease. Table : Useful Diagnostic Tests for Wilson Disease Test Normal Value Wilson Disease Serum 180-350 mg/L ceruloplasmin (18-35 mg/d1) * Low in 85% * Present in 99% KF rings Absent - If neurologic or psychiatric symptoms present. * Present in 30-50% - in hepatic presentation and presymptomatic state * Urinary copper excretion is increased 24-h urine Cu 0.3-0.8 mmol - >1.6intn ol(>100mg) in symptomatic patients * 0.9 to > mmol (60 to > 100 mg) - in presymptomatic patients Liver Cu 0.3 -- 0.8 mmol/g * Liver copper is increased (20-50 mg) tissue * > 3.1 mmol (200 mg) Haplotype analysis 0 Matches 2 Matches Treatment Zinc is the treatment of choice for Wilson diseaseQ. It produces a negative copper balance - By blocking intestinal absorption of copper - By inducing hepatic metallothionein synthesis which sequesters additional toxic copper. Table : Recommended Anticopper Treatments for Wilson Disease Disease Status First Choice Second Choice Initial hepatic manifestations Zinc Trientine and zinc Trientine and zinc Hepatic transplantation Trientine Penicillainine and zinc Hepatic transplantation Trientine and zinc * Hepatitis or cirrhosis without decompensation * Hepatitis or Cirrhosis with decompensation - Mild - Moderate - Severe Initial neurologic/psychiatric Tetraioinolybdate and zinc Trientine and zinc Maintenance therapy Zinc Trientine Presymptomatic therapy Zinc Trientine Pediatric Zinc Trientine Pregnant Zinc Trientine",Surgery, +e41fa443-a31b-4edc-973f-c29a19504a8b,"A 25 year male presented with high grade fever, headache, neck stiffness, on examination found to have neck rigidity, kernig's sign positive, csf analysis showed neutrophilic predominance, low glucose and limulus amebocyte lysate assay was positive. Which of the following is the likely pathogen?",Staphylococcus aureus,Streptococcus pneumonia,Neisseria meningitides,Listeria monocytogenes,2,single,"The Limulus amebocyte lysate assay is a rapid diagnostic test for the detection of gram-negative endotoxin in CSF and thus for making a diagnosis of gram-negative bacterial meningitis. The test has a specificity of 85-100% and a sensitivity approaching 100%. Thus, a positive Limulus amebocyte lysate assay occurs in viually all patients with gram-negative bacterial meningitis, but false positives may occur. Ref Harrison 20th edition page 1001",Medicine,C.N.S +039cfb73-ed02-48d5-9402-91cb407a1250,Tuberculin test denotes:,Previous or present sensitivity to tubercle proteins,Patient is resistant to TB,Person is susceptible to TB,Protective immune status of individual against TB,0,single,"Ans. is 'a' i.e., Previous or present sensitivity to tubercle proteins(a) Tuberculin test denotes Type IV (delayed) hypersensitivity to tuberculoprotein 0.1 ml (Purified protein derivative) and used in diagnosis of latent tuberculosis.(b) Positive tuberculin test indicates exposure to Mycobacterium tuberculosis in the form of infection or immunization with or without active disease.",Microbiology,Bacteria +eca9d17a-d5bd-4f49-a5e2-09f054da5e57,Tumor suppressor gene p53 prevents carcinoma by?,DNA repair,Cell cycle arrest,Apoptosis induction,All of the above,3,multi,"Ans. is 'd' i.e., All of the above * p53 gene is located on chromosome 17 & acts as molecular policeman that prevents the propagation of genetically damage cell. p53 gene product, i.e. p53 protein is a DNA binding protein in the nucleus, when called into action, it controls the transcription of several other genes.* When there is DNA damage due to irradiation, UV light or mutagenic chemicals, there is rapid increase in p53 levels.* p53 causes :-a) Cell cycle arrest: p-53 induces transcription of p21, a CDK inhibitor. p21 inhibit cyclin DCDK- 4 complex and there is arrest of cell cycle late in G1 phase. This allow time for DNA repair.b) DNA repair: p-53 also helps in DNA repair directly by inducing transcription of GADD 45 (growth arrest and DNA damage). GADD 45 encodes a protein that is involved in DNA repair.* If DNA damage is repaired successfully, p53 activate MDM-3 and this MDM-3 induce degradation of p-53 - Relieve in cell cycle block. If DNA damage cannot be successfully repaired, p53 induces apoptosis by inducing the activation of apoptosis inducing gene BAX. So p-53 prevents replication of cell with defective DNA - p53 functions as a critical gatekeeper against formation of cancer. Mutation in p53 leads to carcinogenesis because of loss of above protective mechanism.",Pathology,Neoplasia +6ad43210-4f47-46e0-9336-3f713731bab5,Most specific enzyme for mi is:,Cpk-MM,Cpk-MB,Cpk-BB,LDH,1,single,"Ans: bRef: Harrison, 16thed, p. 1450",Medicine,C.V.S. +71c50ecc-ce9f-452b-9637-20990cc519da,Which of the following is not a part of fatty acid synthase Complex?,Ketoacyl reductase,Enoyl reductase,Acetyl-CoA carboxylase,Ketoacyl synthase,2,single,"Ans. C. Acetyl-CoA carboxylase(Ref: Harper 31/e page 217)Fatty Acid Synthase (FAS) Multienzyme ComplexThe complex is a homodimer of two identical polypeptide monomers in which six enzyme activities and the acyl carrier protein (ACP)ACP contains the vitamin pantothenic acid in the form of 4'-phosphopantetheineX-ray crystallography of the three-dimensional structure, shown that the complex is arranged in an X shapeAcetyl-CoA carboxylase is not a part FAS ComplexQ.",Biochemistry,Lipids +058349ad-8cad-49e2-bd7d-1adefceb3093,All of the followings are responsible for Right shifting of O2 saturation curve; except:,Hypoxia,Hypocapnia,Increase temp,"Increased 2,3 DPG",1,multi,"Ans. b. HypocapniaThe factors shift the curve to the right:Increased CO2 (Bohr effect)Increased hydrogen ion (decrease pH)Increased temperatureIncreased 2,3-bisphosphoglycerate (2,3-BPG)Shifts in Hb-O2 dissociation curveStored blood loses 2, 3-bisphosphoglycerate, causing a left shift in the curve, while hypoxia stimulates the production of 2,3-bisphosphoglycerate, thereby causing a right shift.Extra MileRight shiftAcidCO2 Exercise2, 3-BPGAltitudeTemperatureMnemonics: ACE BATs right handed",Physiology,Respiratory System +de4708cf-da2a-43fb-9096-ea14fe9532c4,"A nineteen year old female with short stature, wide spread nipples and primary amenorrhoea most likely has karyotype of –","47, XX + 18",46XXXY,"47, XXY",45 XO,3,single,,Pediatrics, +08c323e3-034d-4a3c-8c75-7eae5ba43778,True about rash of chickenpox,Centripetal,Deep Seated,Affectspalm,Slow to evolute,0,multi,"Rash of chicken pox : Superficial, unilocular, centripetal, pleomorphic, symmetrical, affects flexor surfaces and axilla, spares palms and sales, has inflammation around, rapid evolution and dew-drop on rose petal appeamace.",Social & Preventive Medicine, +52ba5acd-dd2c-49ce-89d1-9a848b0f775f,All are true about Klumpke's paralysis except,Claw hand is never seen,Intrincsic muscles of hand are paralysed,Horner's syndrome can be associated,Involves lower trunk of brachial plexus,0,multi,"Klumpke's paralysis + +C8 and T1 roots involved. +Intrinsic muscles of hand paralysed. +Wrist and finger flexous are weak. +Unilateral Horner's syndrome. +Claw hand is seen.",Orthopaedics, +5e1e3fb1-06ae-47f7-afb4-6daa01fe92c4,Hyperglycemia is seen in all except?,Cirrhosis,Myotonic dystrophy,Lipodystrophy,Sarcoma,3,multi,"In cirrhosis, blood sugar is elevated, due to reduced uptake of sugar by liver. In myotonic dystrophy and lipodystrophy, there is tissue insensitivity to insulin leading to hyperglycemia. Sarcoma especially large retroperitoneal sarcomas secrete IGF-2 and lead to hypoglycaemia.",Medicine,Diabetes Mellitus +10f7eb85-ebda-4224-bb87-18914bc64576,LSD was introducd by,albe hoffman,delay and deniker,John F Cade,Egaz Moniz,0,single,"Hallucinogens * ALSO CALLED AS Psychotomimetic/ Psychedelic * FIRST Man made HALLICINOGEN ==================== Albe Hoffman * MECHANISM OF ACTION Lysergic acid diethylamide=======serotonin * Introduced by Albe Hoffmann and he himself experienced its effects * Phencyclidine =======NMDA antagonist * Also known as angel dust * Related to ketamine=== dissociative anesthetic * Has anti-suicidal propey Ref.Kaplon and Sadock, synopsis of psychiatry, 11 th edition, pg no.649",Psychiatry,Substance abuse +4a5a1278-2ae8-4808-8548-29acfb03ef49,Decreased basal metabolic rate is seen in,Obesity,Hypehyroidism,Feeding,Exercise,0,single,"The remaining options utilise energy. Basal metabolic rate and obesity The basal metabolic rate accounts for about 60 to 75% of the daily calorie expenditure by individuals. It is influenced by several factors. BMR typically declines by 1-2% per decade after age 20, mostly due to loss of fat-free mass, although the variability between individuals is high. Ref guyton and hall textbook of medical physiology 12/e pg843",Physiology,General physiology +1044a6a2-d3bc-430c-8d3b-972ba533c2a8,"A 70 kg athlete is taken for surgery, due to unavailability of vecuronium, succinylcholine is given repeatedly (>640mg). He now is unable to take breath and his lower limbs are paralyzed. What will be cause?",Phase II block,Succinylcholine causes muscle paralysis due to fasciculation,Hidden muscle dystrophy,Pseudo cholinesterase deficiency,0,single,"Prolonged apnea due to repeated dosing with succinylcholine is due to it entering phase 2 block. Phase I depolarizing block is preceded by muscle fasciculation. During paial neuromuscular block, phase I depolarizing block is characterised by: No fade during repetitive stimulation (tetanic or TOF) No post tetanic facilitation (potentiation) Rapid hydrolysis by butyrylcholinesterase (plasma cholinesterase) terminates phase I depolarizing blockade of succinylcholine. Phase II block resembles non depolarizing block, is characterised by: Fade during repetitive stimulation (tetanic or TOF) Post tetanic facilitation (potentiation) Phase II block can be antagonized by administering a cholinesterase inhibitor like neostigmine.",Anaesthesia,Neuromuscular Blocker +a6ae10e2-80ca-44a2-bfca-5442cdf0d3fd,Inheritance of ABO blood group is:,X-linked inheritance,Recessive inheritance,Mitochondria] inheritance,Codominance,3,single,"Ans. d. Codominance (Ref Robbins 9/e p140; Ganong 23/e p527)Inheritance of ABO blood group is Codominance.""Although Mendelian traits are usually described as dominant or recessive, in some cases both of the alleles of a gene pair contribute to the phenotype--a condition called codominance. Histocompatibility and blood group antigens are good examples of codominant inheritance. ""--Robbins 9/e p140Codominance is a relationship between two versions of a gene, individuals receive one version of a gene, called an allele, from each parent. If the alleles are different, the dominant allele usually will be expressed, while the effect of the other allele, called recessive, is masked. In Codominance, however, neither allele is recessive nor are the phenotypes of both alleles are expressed, e.g., ABO blood group, where in AB blood group both A and B are dominant.""Codominance: When both alleles of a gene pair contribute to the phenotype. E.g., Blood group AB.ABO Blood Group SystemA and B antigens of the ABO blood group system are glycoproteins present on the RBC membrane.H substance is the immediate precursor on which A and B antigens are added.H substance is formed by the addition of fucose to the glycolipid or glycoprotein backbone.The subsequent N-acetyl glucosamine creates the A antigen, while the addition of galactose produces the B antigen.Bombay phenotypeIndividuals with the rare Bombay phenotype (hh) do not express the H antigen (also called the H substance), the antigen, which is present in blood group OQ.As a result of the absence of the H antigen they cannot make either the A antigen or the B antigenQ.These individuals have antibodies not only against A and B antigens but also against the H antigenQ.",Pathology,Mendelian Disorders: Single-Gene Defects +a9ca8f83-da91-4a09-8b5d-0ecb5748a6ae,Which is not ketogenic,Leucine,Lysine,Methionine,Tryptophan,2,single,"Ans. is 'c >d' i.e., Methionine > TryptophanLeucine and lysine are purely ketogenic.Trypotophan is not purely ketogenic, but it is ketogenic along with glucogenic. Methionine is purely glucogenic.",Biochemistry, +fdd78ece-f205-448d-90cd-4c70fc33aab7,Glossodynia is,Pain in the tongue,Burning of the tongue,Swelling of the tongue,White patch on tongue,0,single,,Pathology, +9e813b84-1465-47c4-bce4-d4b9d21aae66,All are the branches of internal iliac artery except:,Obturator artery,Middle rectal artery,Femoral artery,Internal pudendal artery,2,multi,"Ans. C. Femoral arteryThe internal iliac artery supplies the walls and viscera of the pelvis, the buttock, the reproductive organs, and the medial compartment of the thigh.It arises at the bifurcation of the common iliac artery, opposite the lumbosacral articulation, and, passing downward to the upper margin of the greater sciatic foramen, divides into two large trunks, an anterior and a posterior.The anterior division gives rise to these arteries:a. Umbilical artery (in fetus),b. Obturator artery,c. Vaginal artery,d. Superior &inferior vesical artery,e. Uterine artery,f. Middle rectal artery,g. Internal pudendal artery,h. Inferior gluteal artery.The posterior division gives rise to the superior gluteal, iliolumbar, and lateral sacral arteries.",Anatomy,Abdomen & Pelvis +e7a593de-77a8-46f8-b876-3a8186b36abc,SACD (Subacute combined degeneration of cord) is feature of which vitamin deficiency?,Vitamin A,Vitamin B6,Vitamin B9,Vitamin B12,3,single,"Ans. d (Vitamin B12) (Ref. H - 17th/pg. Table 71-1).Vitamins: Major FunctionsVITAMINFUNCTIONVitamin AA component of visual pigment; Maintenance of specialized epithelia; Maintenance of resistance to infectionVitamin DFacilitates intestinal absorption of calcium and phosphorus and mineralization of boneVitamin EMajor antioxidant; scavenges free radicalsVitamin KCofactor in hepatic carboxylation of procoagulants--factors II (prothrombin), VII, IX, and X; and protein C and protein S.Vitamin B1 (thiamine)As pyrophosphate, is coenzyme in decarboxylation reactions. Converted to coenzymes flavin mononucleotide and flavin adenine dinucleotide, cofactors for many enzymes in intermediary metabolismNiacinIncorporated into nicotinamide adenine dinucleotide (NAD) and NAD phosphate, involved in a variety of redox reactionsVitamin B6 (pyridoxine)Derivatives serve as coenzymes in many intermediary reactionsVitamin B12Required for normal folate metabolism and DNA synthesis; Maintenance of myelinization of spinal cord tractsVitamin CServes in many oxidation-reduction (redox) reactions and hydroxylation of collagenFolateEssential for transfer and use of 1-carbon units in DNA synthesisPantothenic acidIncorporated in coenzyme ABiotinCofactor in carboxylation reactionsVitamin B12.introductionStored primarily in the liver. Very large reserve pool (several years). Synthesized only by microorganisms. Found only in animal products.FunctionCofactor for homocysteine methyltransferase (transfers CH3 groups as methylcobalamin) and methyimalonyl-CoA mutase. Abnormal myelin is seen in B12 deficiency, possibly due to | methionine or | methylmalonic acid (from metabolism of accumulated methylmalonyl-CoA).Causes of deficiencyVitamin B12 deficiency is usually caused by malabsorption (sprue, enteritis, Diphyllobothrium latum), lack of intrinsic factor (pernicious anemia, gastric bypass surgery), or absence of terminal ileum (Crohn's disease). Use Schilling test to detect the etiology of the deficiency.Deficiency featuresMacrocytic, megaloblastic anemia; neurologic symptoms (optic neuropathy, subacute combined degeneration, paresthesia); glossitis.PRINCIPAL CLINICAL FINDINGS OF VITAMIN MALNUTRITIONNutrientClinical findingDietary level per day associated with overt deficiency in adultsContributing factors to deficiencyThiamineBeriberi:Neuropathy,Muscle weakness and wasting, Cardiomegaly,Edema,Ophthalmoplegia,Confabulation<0.3 mg/1000 kcalAlcoholism, chronic diuretic use, hyperemesisRiboflavinMagenta tongue,Angular stomatitis,Seborrhea,Cheilosis<0.6 mg-NiacinPellagra:Dermatitis: Pigmented rash with silvery/ varnish-like scales of sun-exposed areas.Bright red beefy tongue.DiarrheaDementiaApathyDisorientationDeath (if untreated)<9.0 niacin equivalentsAlcoholism, vitamin B6 deficiency, riboflavin deficiency, tryptophan deficiencyVitamin B6SeborrheaGlossitisConvulsions (especially in neonates),Neuropathy,Depression,Confusion,Microcytic anemia<0.2 mgAlcoholism, isoniazid (most common cause)FolateMegaloblastic anemia,Atrophic glossitis,Depression, -Homocysteine<100 mg/dAlcoholism, sulfasalazine, pyrimethamine, triamtereneVitamin B12Megaloblastic anemia,SACD:Loss of vibratory and position sense, Abnormal gait,Dementia, impotence,Loss of bladder and bowel control, -homocysteine, -methylmalonic acid.<1.0 mg/dGastric atrophy (pernicious anemia), terminal ileal disease, strict vegetarianism, acid reducing drugs (e.g., H2 blockers)Vitamin CScurvy:petechiae, ecchymosis, coiled hairs, inflamed and bleeding gums, joint effusion, poor wound healing, fatigue, perifollicular hemorrhages, subperiosteal hematoma (Woody leg).<10 mg/dSmoking, alcoholismVitamin AXerophthalmia, night blindness, Bitot's spots,Follicular hyperkeratosis,Impaired embryonic development, Immune dysfunction<300 mg/dFat malabsorption, infection, measles, alcoholism, protein-energy malnutritionVitamin DRickets: skeletal deformation, rachitic rosary, bowed legs; osteomalacia<2.0 mg/dAging, lack of sunlight exposure, fat malabsorption, deeply pigmented skinVitamin EPeripheral neuropathy,Spinocerebellar ataxia,Skeletal muscle atrophy, retinopathyNot described unless underlying contributing factor is presentOccurs only with fat malabsorption, or genetic abnormalities of vitamin E metabolism/transportVitamin KElevated prothrombin time, bleeding<10 mg/dFat malabsorption, liver disease, antibiotic use",Biochemistry,Vitamins and Minerals +dfa84cb3-c018-4049-92c0-9facb1fedf72,Main aim for treatment of TB ?,Prevention of complication,Prevention of disease transmission,Complete clinical cure,Complete bacteriological cure,3,single,"Ans. is 'd' i.e., Complete bacteriological cure Chemotherapy is indicated in every case of active tuberculosis. The objective of treatment is cure-that is, the elimination of both the fast and slowly multiplying bacilli from the patient's body. The effects of chemotherapy are judged not by the anatomic healing of lesions, but maily by the elimination of bacilli from the patient's sputum.",Social & Preventive Medicine, +d6cbcff3-4fd8-472d-8749-874eb7e6676f,"All of the following are functions of CD 4 helper cells, except-",Immunogenic memory,Produce immnoglobulins,Activate macrophages,Activate cytotoxic cells,1,multi,"CD4 T cells are helper T cells. They stimulate B cells to produce antibodies, release of cytokines and are responsible for memory. They do not directly produce any antibody by their own. Basic Pathology, Robbins. Page no.:101",Pathology,General pathology +cd1c6dc6-e13e-46e4-a1bf-0e58f335fcab,"A 35-year-old man with duodenal stump leak after partial gastrectomy is receiving central parenteral nutrition containing the standard D25W, 4.25% amino acid solution. Which is TRUE of essential fatty acid deficiency seen after hyperalimentation?",It occurs if soybean oil is given only once weekly.,It is usually noted at the end of the first week.,It causes dry scaly skin with loss of hair.,It is accompanied by hypercholesterolemia.,2,multi,"Essential fatty acid deficiency usually occurs if hyperalimentation is extended for more than 1 month and when soybean oil is not administered at least twice a week. There is a decrease in linolenic, linoleic, and arachidonic acids and an increase in oleic and palmitoleic acid. In addition to the skin changes, there may be poor wound healing, increased susceptibility to infection, lethargy, and thrombocytopenia. It is characterized by a triene-to-tetraene ratio >0.4.",Surgery,"Wounds, Tissue Repair & Scars" +6bdc5753-1ecc-4c7f-9b13-c12a98cc27b6,Sacrum becomes a single bone at ___ years of age.:-,15,25,30,40,1,single,"Sacrum The five sacral veebrae are separated by cailage until pubey. Later, fusion of epiphyses takes place and ossification of interveebral discs extend from below upwards. Sacrum becomes single bone at 21-25 years of age. Sometimes, it leaves a gap between S1 and S2, until 32 years called as 'lapsed union'.",Forensic Medicine,Human identification +e2fd6f27-7d7f-4716-aa70-ecba8b9eabad,Which is True about dobutamine?,Dobutamine decreases peripheral resistance,Acts on D1 and D2 receptors,Decrease kidney circulation,Has no effect on coronary circulation,0,multi,Ref: HL Sharma 3rd ed pg no: 177 Dobutamine is a derivative of dopamine and is a selective beta1 agonist. It causes an increase in the force of contraction without a significant increase in hea rate. It also produces some vasodilatation by stimulating the beta 2 receptors. It doesn't decrease kidney circulation.,Pharmacology,Autonomic nervous system +530064a0-72a7-4912-a2f0-3d7c504b57a5,Which of the following statements about cutaneous shunt~vessels is true?,Perform nutritive function,Have role in thermoregulation,Not under the control of autonomic nervous system,These vessels are evenly distributed throughout the skin,1,multi,"These are low resistance connection between the aerioles and veins bypassing the capillariesAbundantly innervated by sympathetic nerve fibresEspecially found in skin of fingers, toes and earlobes where they are involved in the thermoregulationRef: Medical Physiology Indu Khurana 2015 edition page No: 237",Physiology,Cardiovascular system +69f3e303-e24a-46df-81de-e7caa5bf29db,Volume of infarcted area in acute myocardial infarction (AMI) can be detected by -a) ECHOb) ECGc) Levels of CPKMBd) Thallium scan,b,c,ac,ad,3,multi,,Medicine, +5d7fcdca-98b9-43e6-bfed-edf8891c9ac9,Blow out fracture of orbit commonly produces,Deviation of septum,Retinal haemorrhage,Fracture of nasal bones,Fracture of floor of orbit,3,single,"(D) Fracture of floor of orbit # Blow out fracture of orbit; Intraorbital haemorrhages, proptosis, paralysis of extrinsic muscles and fracture of the floor of the orbit are not infrequent following a blunt injury to the orbital region> Proptosis develops due to reactive oedema and intra-orbital haemorrhage.> Partial or complete ophthalmoplegia occurs as a result of injury to the muscles or due to profuse orbital oedema.> In the fracture of the floor of orbit, the eyeball is depressed into the maxillary antrum & the inferior rectus and inferior oblique muscles are entrapped causing diplopia.> There occurs limitation of upward gaze, and downward gaze may also be reduced because of the pinched inferior rectus muscle in between the chips of the broken bone> A positive forced duction test, downwards and inwards displacement of the globe and radiological evidence virtually confirm the diagnosis of blowout fracture with incarcerated orbital tissue.> The fracture of the base of skull implicates optic foramen and may cause optic atrophy or pulsating exophthalmos.> The fracture of the optic canal is characterised by a wound at the lateral part of the eyebrow, loss of direct homolateral pupillary reaction and hemianopic field defects.> The patient may suffer from epistaxis and varying periods of unconsciousness.> The pallor of the optic disc may be noticed 2-3 weeks after the injury. Serial radiological tomograms taken at one minute intervals may confirm the diagnosis.",Ophthalmology,Miscellaneous +57b5b287-ca45-4e1b-be8f-a09a847fb97f,Drug used in cancer chemotherapy induced vomiting is?,Aprepitant,Dexamethasone,Ondansetrin,All of the above,3,multi,"Ondansetron is drug of choice for chemotherapy induced vomiting Dexamethasone,lorazepam and aprepitant are also used for chemotharapy induced vomiting.( ref KDT 7/e p876)",Pharmacology,Gastrointestinal tract +0ff5d474-8b83-4e08-a720-1546fc5dee6f,NOT a cause of granular contracted kidney ?,Diabetes mellitus,Chronic pyelonephritis,Benign nephrosclerosis,Chronic glomerulonephritis,0,single,"Ans. is 'a' i.e., Diabetes mellitusCauses of Granular contracted kidney ?Chronic glomerulonephritis (symmetric)Chronic pyelonephritis (asymmetric) o Benign Nephrosclerosis (Symmetric) Sometimes, Diabetes too can cause granular contracted kidney",Pathology, +9a3e2aa9-a4b3-4745-850f-b113a6ec519f,Anticoagulant of choice for prophylaxis of venous thromboembolism in patient with cancer surgery is:-,Heparin sulfate,Protamine sulfate,Low molecular weight heparin,Warfarin,2,single,Venous Thromboembolism- precipitating factor -estrogen in females PREVENTION OF Venous Thromboembolism AMONG HOSPITALIZED PATIENTS Condition Prophylaxis High risk non - ohopedic surgery Unfractionated heparin Low molecular weight heparin (LMWH) Cancer surgery(associated thromboembolism ) LMWH Major ohopedic surgery Warfarin LMWH Aspirin Factor Xa inhibitors (Apixaban) Dabigatran Medically ill patients during hospitalization Unfractionated heparin LMWH Medically ill patients after hospitalization Betrixaban,Pharmacology,Hematology +8141bbae-712a-4c78-b66f-c742fe4eea44,Which of the following does not belong to dystrophin-glycoprotein complex,Perlecan,Dystrophin,Dystroglycan,Sarcoglycan,0,single,"Perlecan is a large extracellular matrix proteoglycan that plays a crucial role in tissue development and organogenesis. Dystrophin-glycoprotein complex The dystrophin-glycoprotein complex provides a structural link between the cytoskeleton of the muscle cell and the extracellular matrix, which appears to stabilize the sarcolemma, adds strength to the muscle by providing a scaffolding for the fibrils and prevents contraction-induced injury(rupture). Organisation of DG complex Dystrophin connects F-actin to the transmembrane protein b-dystroglycan smaller proteins called syntrophins. This b-dystroglycan, in turn connects to the merosin subunit of laminin 211 in the extracellular matrix a-dystroglycan The dytroglycans are also associated with a complex of four transmembrane glycoproteins, a-,b-,g- and d- sarcoglycan and sarcospan. Ref: Ganong's Review of Medical physiology 25th edition Pgno: 103",Physiology,General physiology +0872023a-b442-4c33-8fb4-e916e3cbc681,Patient came to the OPD with recent onset photphobia within 24 hours and sloughing corneal ulcer. There is greenish ulcer base. Which of the following can be the causative organism:,Acanthamoeba,Nocardia,Pseudomonas,Herpes,2,single,Pseudomonas Ulcer Rapid onset & Greenish ulcer base Produce biofilm that cause resistance MC cause of infection in contact lens users Nocardia ulcer resembles fungal ulcer in its characteristics.,Ophthalmology,Diseases of Cornea +b369df7e-9844-445d-911d-e14306aaf867,Culture media used for leptospira,MYPA agar,BYCE agar,EMJH Medium,Skirrow's medium,2,single,"Culture media for leptospira is EMJH media, Korthof's, Stuart's and fletcher's media.",Microbiology, +dd9154c8-18f3-40ee-9309-5be6f26e6818,The advantage of using a tooth positioner as a retainer is :,Final setting of occlusion,Used in uncooperative patients,Esthetics,Gingival margin inflamed during orthodontic treatment,3,single,"The use of a tooth positioner rather than final settling archwires has two advantages:  + +It allows the fixed appliance to be removed somewhat more quickly than otherwise would have been the case (i.e., some finishing that could have been done with the final archwires can be left to the positioner) +It serves not only to reposition the teeth but also to massage the gingiva, which is almost always at least slightly inflamed and swollen after comprehensive orthodontic treatment. The gingival stimulation provided by a positioner is an excellent way to promote a rapid return to normal gingival contours. + +As a general rule, a tooth positioner in a cooperative patient will produce any changes it is capable of within 2-3 weeks. Final (post-treatment) records and retainer impressions can be taken 2 or 3 weeks after the positioner is placed. Beyond that time, if the positioner is continued, it is serving as a retainer rather than a finishing device-and positioners, as a rule, are not good retainers. +Contemporary orthodontics- proffit 4th edition page 614",Dental, +0c1fe138-cff1-4040-a4cc-b16466e99a3e,Cryosurgery is used in treatment of cervical intraepithelial neoplasia. It is appropriate in all except,Transformation zone lying entirely on ectocervix.,Smooth cervical surface without deep crevices.,Cervical intraepithelial neoplasia -3.,CIN limited to two quadrants of the cervix.,2,multi,"Cryosurgery is generally not favoured for treatment of CIN-3 due to higher rates of disease persistence following treatment, and lack of histologic specimen to exclude occult invasive cancer.",Gynaecology & Obstetrics, +a67701a3-f954-488a-875d-12f4fd7ef127,"In respiratory and GIT infections, which is the most affected immunoglobulin -",IgA,IgG,IgM,IgD,0,single,"IgA Antibody:- IgA is the second most abundant antibody (2nd highest for DHS). It is of two types: Serum IgA: Predominantly in monomeric form. Secretory IgA (SIgA): It is dimeric (valency four); Secretory IgA is responsible for Mucosal /local immunity. IgA also exist in two subclasses/isotypes: IgA1 is mainly found in serum. IgA2 predominates in secretions. IgA (Secretory IgA) is selectively concentrated in secretions and on mucous surfaces forming an antibody paste and is believed to play, an impoant role in local immunity against respiratory and intestinal pathogens. It protects the mucous membranes against microbial pathogens. It serves an impoant effector function at mucous membrane surfaces, which are the main entry sites for most pathogenic organisms.",Microbiology,"Immunology Pa 1 (Immune Response, Antigen-Antibody Reactions, Hypersensitivity, Structure of Immune System, Immunodeficiency Disorders)" +745dd5aa-461f-4928-9b6a-6bcd597d685e,All are risk factors of squamous cell carcinoma except,Ultraviolet radiation,Tar,Tobacco,HPV 18,3,multi,"Ultraviolet radiation, chronic inflammation, and chemical carcinogens (arsenicals, tar) and infection with HPV 5 and 16.There is also evidence that current and previous tobacco use doubles the relative risk of SCC.Ref: Bailey and Love, 27e, page: 606",Surgery,General surgery +dc30cf9f-3905-4ae1-ac1a-97312f1567a8,Judge can ask questions: Punjab 07,During cross-examination,Examination-in-Chief,Any time during trial,Re-examination,2,single,Ans. Any time during trial,Forensic Medicine, +96952c56-c9f6-43f2-a638-122711492aa0,Which is not an aryl phosphate -,Parathion,TIK-20,Malathion,Paraoxon,2,single,"Organophosphate poisoning is the most common poisoning in India followed by aluminium phosphide. Organophosphorus compounds are classified as 1. Alkyl compounds -- such as tetraethyl pyrophosphate (TEPP), hexa ethyl tetraphosphate (HETP), octa methyl pyrophosphate (OMPA), malathion etc. 2. Aryl compounds -- such as parathion, chlorothion, diazinon (Tik-20), paraoxon etc. REF;THE SYNOPSIS OF FORENSIC MEDICINE:KS NARAYANA REDDY;28th EDITION;PAGE NO 291",Forensic Medicine,Poisoning +59165dbb-66fe-4ccd-bd9f-26c60ca3c161,"A female, Lalita, aged 26 yrs accidentally takes 100 tablets of paracetamol. What is the treatment of choice -",Lavage with charcoal,Dialysis,Alkaline diuresis,Acetylcysteine,3,multi,"Ans. is 'd' i.e Acetylcysteine ""In cases of moderate to severe poisoning N acetylcysteine (Mucomyst) should be given orally. It is most effective when given within 16-24 hours of overdosage. It prevents hepatic damage"". - Parikh",Unknown, +8f26f931-d9ad-4446-8735-8a2a0891cd89,True for Bochdalek hernia:,Commonly occurs on the right side,Congenital disorder,Anterior position,Breathing difficulty in newborns from day 2-3,1,multi,"Ans. B. Congenital disorderBochdalek hernia is the other name for congenital diaphragmatic hernia. It is a posterior (and not anterior) congenital defect of the diaphragm and is primarily due to lack of closure of the pleuroperitoneal cavity between the eighth and tenth week of embryonic life. Majority of cases occur on the left side. Affected newborns frequently present with breathing difficulty, particularly within the first few hours of life.",Pediatrics,New Born Infants +7fc5f783-e5de-47e8-be02-0b2e2d2cf123,Hypercalcemia is most commonly associated in which of the following cancers?,Renal cell cancer,Carcinoma stomach,Squamous cell carcinoma of lung,Hepatocellular carcinoma,2,single,"The most common neoplasm associated with hypercalcemia is squamous cell carcinoma. Other tumors often associated with paraneoplastic hypercalcemia are carcinomas of the lung, kidney, breast and ovary. Paraneoplastic hypercalcemia is caused by parathyroid hormone-related protein (PTHRP) secreted by these tumours. PTHRP increases bone resorp-tion and renal calcium uptake, while inhibiting renal phosphate transpo, effects that raise serum calcium levels.",Pathology,General Concepts +6cbef59c-cfd7-44a3-be03-2dcde2226526,Ethanol is given in methyl alcohol poisoning because:,It inhibit alcohol dehydrogenase,It inhibit aldehyde synthetase,It binds 100 times stronger than methanol,None,0,multi,Ethanol is preferentially metabolised by alcohol dehydrogenase over methanol.At a concenteration 100mg/DL in blood it saturates alcohol dehydrogenase over methanol &retads methanol metabolism. This reduces formation of formaldehyde & formic acid. Ref KD Tripathi 8th ed.,Pharmacology,Central Nervous system +a37fa7cc-898e-46b1-8909-8ea8c34e5761,"A term born 6-month-old, lethargic blonde infant, presented with multiple episodes of vomiting, poor feeding, convulsions and severe psychomotor retardation. O/E: - Microcephaly with prominent maxillae and widely spaced teeth Blue iris Seborrheic or eczematoid rash Spasticity, hyperreflexia, and tremor Musty odour of urine MRI brain All of the following statements are true about the disease except: -",May be caused due to the deficiency of cofactor tetrahydrobiopterin BH4,Tandem mass spectrometry is useful in the diagnosis.,The complications can't be prevented by restricting the use of the involved amino acid in diet.,Pegvaliase can be given in adults suffering from this.,2,multi,"This is a case of Phenylketonuria. Autosomal recessive Deficiency of Phenylalanine hydroxylase May also be due to deficiency of cofactor tetrahydrobiopterin BH4 Leads to hyperphenylalaninemia - Impairs brain development Normal at bih Becomes symptomatic later on in life Present with seizures, hypopigmentation of skin and hair, mental retardation. Complications can be avoided by restricting phenylalanine intake in diet. Diagnosed by: - Guthrie test Tandem mass spectrometry- measure amino acid Immunoassays MRI image shows affected periventricular/ parieto-occipital white matter (due to hypomyelination). Pegvaliase can be given in adults.",Unknown,Integrated QBank +7d0da418-4874-4cf7-b692-7ac48dae2708,PK reactio detects -,IgG,IgA,IgE,IgM,2,single,"The Prausnitz-Kustner test (PK test, Prausnitz-Kustner reaction) is an immunologic test formerly used by physicians to determine if a patient has an allergic reaction to a specific antigen i.e, IgE Ref: Ananthanarayan & Parkers textbook of microbiology 9th edition pg:98",Microbiology,Immunology +9fd65ddc-1d99-4f2a-9cde-eb506e3f3aef,Most common cause of facial nerve palsy:,Idiopathic Bell's palsy,Herpes zoster oticus,Mastoid surgery,Chronic suppurative Otitis media,0,single,"Facial Nerve - Course Intracranial pa 15-17mm Intra temporal pa Meatal segment 8-10mm Labyrinthine segment 4.0mm Tympanic / horizontal segment 11.0mm Mastoid / veical segment 13.0mm Extracranial pa Branches Greater superficial petrosal nerve 1st branch Nerve to stapedius Chorda tympani Communicating branch Posterior auricular nerve Muscular branches, peripheral branches Causes of facial nerve palsy Most common cause is idiopathic bell's palsy paralysis acute onset Bell's palsy idiopathic, Ipsilateral lower motor neuron palsy Loss of forehead and brow movements Inability to close eyes drooping of eyelids Loss of nasolabial folds, drooping of lower lip herpes zoster oticus/Ramsay hunt syndrome herpes zoster oticus/Ramsay hunt syndrome Mastoid surgery is the most common iatrogenic cause of facial nerve palsy.",ENT,FMGE 2018 +58414fd5-9a46-4f3e-9877-99124ced85fd,Sinus of Morgagni is between:,Middle constrictor and inferior constrictor,Middle constrictor and superior constrictor,Superior constrictor and skull,None of the above,2,multi,"Sinus of Morgagni is a gap between the base of skull and the upper concave border of superior constrictor muscle. +Structures passing through that are: Auditory tube, levator palati muscle, ascending palatine artery and palatine branch of the ascending pharyngeal artery.",Anatomy, +cf27425c-59c2-4d8c-90c9-8fe4c02249df,The typical bone marrow finding in ITP is:March 2004,Absent megakaryocytes,Foam cells,Increased megakaryocytes,Fragmented megakaryocytes,2,single,"Ans. C i.e. Increased megakaryocytesDiagnosis of ITPDespite the destruction of platelets by splenic macrophages, the spleen is normally not enlarged.In fact, an enlarged spleen should lead to a search for other possible causes for the thrombocytopenia.Bleeding time is usually prolonged in ITP patients.Normal bleeding time does not exclude a platelet disorder.Bone marrow examination may be performed on patients over the age of 60 and those who do not respond to treatment, or when the diagnosis is in doubt.On examination of the marrow, an increase in the production of megakaryocytes may be observed and may help in establishing a diagnosis of ITP.An analysis for anti-platelet antibodies is a matter of clinician's preference",Pathology, +eaae9ff2-57e0-4987-91b7-aa9ee8d1d304,The final sugars in intestinal chyme are,Glucose and fructose,Ribose and mannose,Ribose and xylulose,Xylulose and fructose,0,single,,Physiology, +b89a8870-4b04-40a5-b61c-08b2f20f955a,Abdominal ultra-sonography in a 3 year old boy show a solid circumscribed hypoechnoic renal mass. Most likely diagnosis is -,Wilm's tumor,Renal cell carcinoma,Mesoblastic nephroma,Oncocytoma,0,single,"Amongst the given options only Wilms tumor is common at 3 years of age :- +  Wilm's tumor                  → young children (mean 3 years) + Mesoblastic nephroma     → < 1 year + Renal cell carcinoma        → > 40 years + Oncocytoma                    → 25-95 years (mean 65)",Radiology, +602e8406-1778-4b75-9fd9-9fd67000b90e,The term that best describes the nuclear dissolution is,Pyknosis,Karyorrhexis,Karyolysis,None of the above,2,multi,"Pyknosis is condensation of nucleus. +Karyorrhexis is fragmentation of nucleus. +Karyolysis is dissolution of nucleus.",Pathology, +38548ecb-a51d-4c93-bb67-bd82e9d4af79,Which of the following test is used to estimate the amount of fetal maternal hemorrhage:,Coomb's test,Apt test,Liley's spectrophotometer,Kleihauer betke test,3,single,"KLEIHAUER BETKE TEST or ACID ELUTION TEST done to measure the amount of feto maternal hemorrhage usually in an Rh negative mother to calculate the dose of anti D required This method is based on the fact that an acid solution (citric acid phosphate buffer, pH 3.5) elutes the adult but not the fetal hemoglobin from the red cells; can detect as little as 0.2 ml of fetal blood diluted in 5 L of maternal blood. More accurate tests are immunofluorescence and flow cytometry. Schiller test:- Schiller's iodine solution is applied to the uterine cervix under direct vision. Normal mucosa contains glycogen and stains brown, whereas abnormal areas, such as early CIN, do not take up the stain and appear white/yellow. Liley's spectrophotometer: Spectrophotometric analysis of amniotic fluid at optical density difference at 450 nm wavelength to see detion bulge in Rh hemolytic disease.",Gynaecology & Obstetrics,Obstetrics +c8b518b5-9d0e-4188-a73e-4ddcb59365e0,Failure to initiate and maintain spontaneous respiration following birth is clinically k/a –,Birth asphyxia,RDS–Respiratory distress syndrome,Respiratory failure,Pulmonary oedema,1,multi,,Pediatrics, +12e7ce79-1a49-41e2-b806-16ac4daee47d,In volume cycled ventilation the inspiratory' flow rate is set at:,140-160 L/min,110-130L/min,60-100 L/min,30-50L/mm,2,single,"In volume cycled ventilation the inspiratory flow rate is usually set at 60-100 U/min to allow greater expiration time for each breath. Use of high inspiratory flow rate can minimize end inspiratory lung volume and intrinsic PEEP, but it can cause higher peak airway pressures. Volume cycled assist control mode of ventilation is the most commonly used mode of ventilation. Volume targeted modes deliver a preset volume unless a specified circuit pressure limit is exceeded. Its major advantages are capacity to deliver unvarying tidal volumes, flexibility of flow and volume adjustments, and power to ventilate difficult patients. Disadvantages of this type of ventilation are: Unless the airway is well sealed volume cycled modes cannot ventilate effectively and consistently. After the flow rate and profile are set the inflation time of machine is set and remains unresponsive to patients native cycling rythm and flow demands. Ref: Respiratory Emergencies By Stephan Kamholtz, page 413. Critical Care Medicine: The Essentials By John J. Marini, page 134.",Anaesthesia, +19950a16-a96f-42d6-baef-f52950186f2b,Which of the following statement is/are true about Obstructive lung disease ?,"mMRC dyspnea scale ,grade 2 includes shoness of breath when hurrying up inclines","Decreased FEV1/FVC ,TLC is increased",Normal PEFR and decreased in Lung volume,Roflumilast is introduced in management of COPD in Group C,1,multi,"Acute asthma and Emphysema- TLC | due to air trapping Myasthenia gravis- TLC | RV| Obstructive Restrictive | in airflow | Lung volume - Expiratory -Inspiratory COPD Asthma Bronchiectasis ILD Scoliosis NM causes FEV1-| FVC- Normal or | TLC - Normal or | | PEFR Normal | | |PEFR Management of COPD Group A Low Risk, low symptom Bronchodilator Group B Low Risk, High symptom LABA/LAMA, if failed- LAMA+LABA Group C High Risk, Low symptom LAMA+LABA or LABA+ICS Group D High Risk, High symptom LAMA+LABA+ICS | symptoms persists Phosphodiesterase inhibitor (Roflumilast) or Macrolides mMRC DYSPNOEA SCALE (for COPD patients) Grade Impact 1 Incline (Breathlessness on going upstairs) 2 Slow walk than peers (Breathlessness on walking in straight surface) 3 Stops (Keeps on stops walking in between) 4 Too breathless to go out of his house GOLD (Global Initiative for Obstructive Lung Disease) classification",Medicine,COPD and Asthma +6d0376b2-01a1-4bb7-a704-26adee194bf5,Diplopia is not a presenting feature in:,Manifest squint,Paralytic squint,Latent squint,Anisometropia,2,single,"C i.e. Latent squint Hetrophoria or latent strabismus is a condition in which there is a tendency to misalignment of the visual axis, which is corrected by the fusional capacity. Often latent squints give no trouble until the demand of near vision increase the strain. No symptoms arise, perhaps, until after reading or writing for an hour or two when the letter seems to run together'. There diplopia, which is often not appreciated as actual double vision, causes blurring of the print. With effo, blurring is overcome, but eventually this becomes impossible, headache supervenes & the work has to be abandoned. Anisometropia presents with - imperfect binocular vision, amblyopia, squint & diplopia (Basak p-68).",Ophthalmology, +d67a810d-ee8d-4738-8547-b7f42558fe6d,Not a branch of external carotid aery:-,Superior thyroid,Ophthalmic,Lingual,Ascending pharyngeal,1,single,Branch of ECA: - Superior thyroid aery Lingual aery Facial aery Anteriorly Ascending pharyngeal aery medially Occipital aery Posteriorly Posterior auricular aery Superficial temporal aery Maxillary aery Terminal aery Branches of ICA Ophthalmic aery Anterior choroidal aery Anterior cerebral Middle cerebral Posterior communication aery.,Anatomy,JIPMER 2018 +9c40aa07-0f46-43b6-b8c0-82de8d34dc21,Not a marker for hyperparathyroidism is :,Increase in serum calcium,Increase in 24 hour urinary calcium excretion,Decrease in serum calcitonin,Subperiosteal resorption of phalanges,2,single,Answer is C (Decrease in serum calcitonin) : Serum calcitonin is no marker fir hyperparathyroidism. It antagonizes the actions of parathyroid hormone and may be used as a modality of treatment of hypercalcemia secondary to hyperparathyroidism. Markers of hyperparathyroidism (|ed PTH): 1. Increased serum calcium: Stimulates Vit D which causes increased absorption of Ca-H- from gut Increased calcium reabsorption from tubules Increases osteoblastic activity in bones and mobilizes calcium from bone into serum. 2. Decreased serum phosphorus: - PTH acts on tubules to increase excretion of phosphorus. 3. Increased 24 hr urine calcium: - Despite increased reabsorption of Ca++ in renal tubules urinary calcium is increased owing to increased filtration of calcium in glomerular filtrate. 4. Increased Alkaline phosphatase: -Increased resorption of bone leads to compensatory elevation of osteoblastic activity 5. Radiological changes: Subperiosteal resorption of phalanges is characteristic (hand X Rays are always advised) Bone resorption i.e. osteitis fibrosa et cystica (brown tumours) `Salt pepper' or 'pepper pot skull' appearance - Loss of lamina dura,Medicine, +a4be5e8f-df2e-47c3-bd52-8a502b42c096,Which of the following is TRUE about branchial cyst?,Usually seen along the posterior border of the sternocleidomastoid muscle,Mostly arises from 2nd branchial system,Always present in preschool children,Conservatory treatment is indicated,1,multi,"Branchial cleft cysts usually present as a soft cystic mass along the anterior border of the sternocleidomastoid muscle. These lesions are usually recognized in the second or third decades of life. To prevent recurrent infection and possible carcinoma, they should be completely excised, along with their fistulous tracts. First branchial cleft cysts present high in the neck, sometimes just below the ear. A fistulous connection with the floor of the external auditory canal may be present. Second branchial cleft cysts, which are far more common, may communicate with the tonsillar fossa. Third branchial cleft cysts, which may communicate with the piriform sinus, are rare and present low in the neck. Ref: Lustig L.R., Schindler J.S. (2013). Chapter 8. Ear, Nose, & Throat Disorders. In M.A. Papadakis, S.J. McPhee, M.W. Rabow, T.G. Berger (Eds), CURRENT Medical Diagnosis & Treatment 2014.",Surgery, +d7c32d13-4b43-4d79-90aa-6a059cca55e0,"In a UK study, it was found that there were more deaths from asthma than the sale of anti-asthma drugs. This is an example of:",Cohort study.,Case reference study.,Ecological study.,Experimental study.,2,single,,Dental, +1dec7811-ae14-4d26-9470-0c73a4f8d0aa,"Pawaer burr, appearance on laparoscopy is characteristic of :",Endometriosis,Endometroid tumour,Epithelial ovarian tumour,Endometrial cancer,0,single,Endometriosis,Gynaecology & Obstetrics, +f10db7ed-acb5-46dd-9714-e1079a390de9,An adult female patient presents with asthma symptoms for at least 4 days a week and wakes up at night due to breathlessness approximately 2 -3 days in a month. she is classified to be having.,Severe persistent Asthma,Mild persistent Asthma,Intermittent Asthma,Moderate persistent Asthma,1,single,,Medicine, +eef73746-8a75-4d05-befb-67d65f405a3f,Which structure(s) passes behind the inguinal ligament -a) Femoral branch of genitofemoral nerveb) Superficial epigastric arteryc) Psoas majord) Femoral veine) Saphenous vein,abc,ad,bc,acd,3,single,"Structures passing deep to inguinal ligament. +-        Psoas major, Iliacus, pectineus +-        Femoral nerve and vessels +-        Lateral cutaneous nerve of thigh +-        Femoral branch of genitofemoral nerve +-        Lymphatics",Anatomy, +33d9bc14-330b-42c8-b597-0a4388ba22df,I/V contrast is not used in –,HRCT,MRI,IVP,Myelography,3,single,"IV contrast agent is given in :- + + +X-ray based investigation (Radiocontrast agent) :- Radiography (simple x-ray), CT-Scan. +MRI (MR contrast agents). +Ultrasound (USG contrast agents). + + +IVP (intravenous pyelography) as the name suggests, the contrast agent is given intravenously. +In myelography, the contrast agent is injected into subarachnoid space.",Radiology, +241f1ab2-55fa-4a54-8351-6db48a5bcf4a,Patient with persistent diarrhea & hypotension. Diagnosis -,VIPoma,ACTHoma,GRFoma,Glucagnoma,0,single,"Ans. is 'a' i.e., VIPoma Pancreatic Neuroendocrine TumorsTumourBiologicaly active peptide secretedTumour locationMalignant percentageMain symptoms and signsGastrinoma (non b cell tumour)GastrinDuodenum (70%) Pancreas (25%) other sites (5%)60-90o Pain (79-J 00%)o Diarrhoea (30-73%)o GERD (30-35%)o Peptic ulcerInsulinoma (b cell tumour)InsulinPancreas > 99% (Insulinomas are distributed equally on head body and tail of pancrease)< 10o Symptoms of hypoglycemiao Symptoms releive on administration of glucoseVIPOMA (Verner-Morrison syndrome, pancreatic cholera (WDHA)Vasoactive intestinal peptidePancreas 90%40-70o Watery diarrhoea (90-100%)o Hypokalemia (80-100%)o Hypochlorhydriao Dehydration (83%)GlucagonomaGlucagonPancreas 100% (usually occurs singly in pancreatic tail)50-80%o Dermatitis (migratory necrolytic erythema) 67-90%o Glucose intolerance (40-90%)o Weight loss (66 to 96%)o .Anemia(33-85%)o Diarrhoea (15-29%)o Thromboembolism",Medicine,G.I.T. +ba5b5d9c-6e4d-4073-a17a-2cf5247d2c1e,Lamina densa of the basal lamina beneath the epithelium is composed of,Type I Collagen,Type II Collagen,Type III Collagen,Type IV Collagen,3,single,,Dental, +05987701-c793-4312-9f50-17a2420f70e6,Haascheiben cells in epidermis are responsible for?,Touch,Temperature,Pressure,Proprioception,0,single,"Ans. is 'a' i.e., Touch",Skin, +28e5fc62-670b-4c5a-90fe-531e5638943d,Pancytopenia with cellular marrow is seen in all Except:,Megaloblastic anemia,Myelodysplasia,Paroxysmal nocturnal hemoglobinuria,G6 PD deficiency,3,multi,"Answer is D (G-6PD Deficiency) Glucose - 6 - PD Deficiency is not associated with Pancytopenia and a cellular marrow Myelodysplasias, PNH and Megaloblastic anemia (Vit B12,Folate deficiency) may present with Pancytopenia and a cellular marrow.",Medicine, +9e4156f1-ffbe-462c-ab24-8c77494d025c,The following are adverse effects of SSRIs except,Weight changes,Diarrhoea,Delayed ejaculation,Galactorrhea,3,multi,"Galactorrhea is a side effect of hyperprolactinemia is not usually seen with SSRIs Side effects of SSRIs include weight changes, delayed ejaculation,diarrhoea,bleeding , discontinuation syndrome,insomnia. Ref: KD Tripathi 8th ed.",Pharmacology,Central Nervous system +4fce6607-dcce-4b01-bb64-40c4d8cc968a,The germinal cell layer surrounding the oocyte before ovulation is known as :,Zona pellucida,Zona reticularis,Cumulous oophorus,Zona glomerulosa,2,single,"In one area , the granulosa cells are collected together to form a projection into the cavity of graffian follicle. This projection is referred to as the discus proligerus or cumulus oophorus. The ovum itself lies within it.With the exception of area around the discus, the peripheral granulosa cells form a layer only a few cells in thickness.,whereas at the discus the cells are between 12 and 20 layers thick. Reference: Shaw's Textbook of Obstetrics 15th edition page 28",Gynaecology & Obstetrics,General obstetrics +1f10b85e-0f5b-4265-97c2-c4c1b8b13252,Folie a deux is,OCD,Shared delusion disorder,Hysteria,Neurasthenia,1,single,"Shared psychotic disorder or folie a deux, the characteristic feature is the transmission of delusions from ""inducer"" (primary patient), who is the ""originally"" ill patient and suffers from a psychotic disorder to another person who may share the inducer's delusions in entirety or in pa. the person who receives the delusion is usually financially dependent on the inducer person the person who receives the delusion is intellectually less normal and tend to beleive everything that the inducer person tells. both the person who induces and receives the delusion live in close proximity they live in social isolation with less contact with the outside world as they remain together delusion system perpetutes. treament of choice for such patients is seperation of the receiver of delusion from the inducer and the symptom of delusion reduces. Depending on whether the delusions are shared among two, three, four, five and even twelve people, it is called as folie a deux, folie a trios, folie a quatre, folie a cinq and folie a douze. Shared psychotic disorder is mostly observed among people who live in close proximity and in close relationships. Reference: Kaplon and sadock, 11 th edition, synopsis of psychiatry, 11 th edition, pg no. 390",Psychiatry,Personality disorders +df6cb8f2-27d5-41be-a8b1-32cdda520ca6,"After the death of the 78 years old male patient in a hospital , who was suffering from COPD. His relatives entered the hospital with heavy sharp weapons and damaged the hospital propey and stared abusing and beating the doctor as well as his staff, Violence against a medical practitioner is considered as:",Non-cognizable and non-bailable offense,Non-cognizable and bailable offense,Cognizable and bailable offense,Cognizable and non-bailable offense,3,single,"Medical protection act: The Act, covering doctors affiliated to institutions as well as independent practitioners, outlaws attacks against physicians and damage to their propey. Offenders can get a jail term of up to 3 years and a fine of Rs 50,000. As per the medicare service persons and damage to propey in medicare service institutions (Prevention of violence and damage or loss to propey) Act , Violence against a registered medical practitioner is considered as a cognizable and non-bialable offence.",Forensic Medicine,Medical Jurisprudence +c132f6f0-3e32-4a1a-9f86-4f2410c31ab4,Which soft tissue sarcoma commonly gives to bone secondaries: (PGI June 2008),Fibrosarcoma,Liposarcoma,Osteosarcoma,Neurofibroma,2,single,"Ans. C (Osteosarcoma) ""Any cancer can spread to bone ,but in adults more than 75% of skeletal metastases originate from cancers of the prostate,breast,kidney & lung. In children, metastases to bone originate from Neuroblastoma, Wilm's tumor, Osteosarcoma, Ewing sarcoma & Rhabdomyosarcoma""- Robbins7th/1303Sarcoma Metastasizing through lymhatics (causing L.N involvement) LymhosarcomaQ RhabdomyosarcomaQ AngiosarcomaQ Clear cell sarcomaQ Epithelial sarcoma M aligna nt fibrous histiocytomaQ Synovial cell sarcomaQ",Orthopaedics,Osteogenic Sarcoma +6d71ffb5-12aa-46f1-84a3-1849029c6f93,The intermediate host for T. Saginata is:,Man,Cow,Dog,Pig,1,single,Cow,Microbiology, +98dce37e-30db-461f-81bd-8fc6e3c21dca,Which of the following is associated with cough?,Lisinopril,Propranolol,Verapamil,Sodium nitroprusside,0,single,"Ans: a (Lisinopril) Ref: KDT, 6th ed, p. 484* Cough is a side effect of ACE-1.* Bradykinin and substance P seem to be responsible for the cough. Thromboxane antagonism, aspirin, and iron supplementation reduce cough induced by ACE inhibitors. Once ACE inhibitors are stopped, the cough disappears, usually within 4 days.Important points on ACE inhibitorsEnalapril, fosinopril, perindopril, and ramipril are prodrags.Captopril & lisinopril does not require hydrolysis to become active ACE inhibitorEnalapril is converted in the body into enalaprilat. Enalaprilat differs from captopril in that it is an analog of a tripeptide rather than a dipeptide.Adverse effects of ACE inhibitors are cough; hypotension; hyperkalaemia; acute renal failure in patients with bilateral renal artery stenosis, stenosis of the artery to a single remaining kidney, heart failure, or volume depletion owing to diarrhea or diuretics; teratogenicity; skin rash; proteinuria, angioedema and dysguesia.",Pharmacology,ACE Inhibitors +8821ae63-6fe0-4ddd-912e-de5be35752f4,Lowest most level of integration of stretch reflex is at,Cerebral cortex,Lower Medulla,Spinal cord,All of the above,2,multi,,Physiology, +b74d5aca-2c9f-42ed-8184-98c465b7ac61,Wernicke&;s encephalopathy involves,Mammillary body,Thalamus,Frontal lobe,Arcuate fasciculus,0,single,"Wernicke's encephalopathy : The neuropathological lesions are symmetrical and paraventricular, involving the mammillary bodies, the thalamus, the hypothalamus, the midbrain, the pons, the medulla, the fornix, and the cerebellum. Ref: Synopsis of Psychiatry, 11th edition, page 632.",Psychiatry,Substance abuse +824706cb-9571-4bcd-ae12-d893256f0c35,"When VA/Q is infinity, it means",Dead space,The PO2 of alveolar air is 159 mmHg and PCO2 is 40 mmHg,Paial pressure of O2 and CO are equal,Atelectasis,0,single,"-Ventilation / perfusion ratio - normal = 4l/min/5l/min=0.8 - AV/Q= INFINITY - Means Perfusion (Q) to lungs is zero that is blood supply to that area is hindered making it awasted ventilationin those alveoli. - This may be due to pulmonary embolism causing |ed Physiological DEAD space . DEAD SPACE- Some of the air a person breathes never reaches the gas exchange areas but simply fills respiratory passages where gas exchange does not occur, - called so as its not useful for gas exchange. Physiologial / total dead space = anatomical dead space +alveolar dead space (non perfused/ non functioning Alveoli) In Atelectasisventilation: perfusion ratio (VA/Q) iszero. it is is the most common cause of a physiologic shunt.",Physiology,NEET Jan 2020 +95fbb4f0-8b34-4251-b067-03673cedc609,Which is true regarding Savage syndrome?,Receptor defect to gonadotrophic hormones,Sho stature,Ovaries do not contain follicles,FSH is normal,0,multi,"In Savage syndrome, there is receptor defect to gonadotrophic hormones. The clinical features resemble autoimmune disease. Height is normal. The ovaries contain follicles. FSH is raised. Ref: Shaw's Textbook of Gynaecology, 18th edition, p286",Gynaecology & Obstetrics,All India exam +0c4db971-ade8-4aeb-8c6f-0bbcd0212c39,What is the dosage of retinol palmitate for early stages of xerophthalmia: September 2011,90 mg orally on two successive days,100 mg orally on two successive days,110 mg orally on two successive days,120 mg orally on two successive days,2,multi,"Ans. C: 110 mg orally on two successive days Nearly all of the early stages of xerophthalmia can be reversed by administration of a massive dose (2,00,000 IU or 110 mg of retinol palmitate) orally on 2 successive days Xerophthalmia/ Dry eyes Condition in which the eye fails to produce tears. Xerophthalmia caused by a severe vitamin A deficiency is described by pathologic dryness of the conjunctiva and * cornea. The conjunctiva becomes dry, thick and wrinkled. If untreated, it can lead to corneal ulceration and ultimately to blindness as a result of corneal damage. Xerophthalmia is a term that usually implies a destructive dryness of the conjunctival epithelium due to dietary vitamin A deficiency -- a rare condition in developed countries, but still causing much damage in developing countries. Other forms of dry eye are associated with aging, poor lid closure, scarring from previous injury, or autoimmune diseases such as rheumatoid ahritis and SjOgren's syndrome, and these can all cause chronic conjunctivitis. Radioiodine therapy can also induce xerophthalmia, often transiently Xerophthalmia from hypovitaminosis A is specifically due to lack of the hormone-like vitamin A metabolite retinoic acid Treatment can occur in two ways: treating symptoms and treating the deficiency. Treatment of symptoms usually includes use of aificial tears in the form of eye drops, increasing the humidity of the environment with humidifiers, and wearing wrap around glasses when outdoors. Treatment of the deficiency can be accomplished with a Vitamin A or multivitamin supplement or by eating foods rich in Vitamin A. Treatment with supplements and/or diet can be successful until the disease progresses as far as corneal ulceration, at which point only an extreme surgery can offer a chance of returning sight.",Social & Preventive Medicine, +3812332f-67f7-4c05-860e-c6c0e51325bc,Which among the following organisms causes Buruli ulcer?,M. Marinum,M. Ulcerans,M. kansasii,M. Smegmatis,1,single,"Ans. B. M. UlceransM. Ulcerans causes Buruli ulcer.* It starts as a nodule or papule which progresses further to form a shallow necrotic ulcer.* Children (5-15 years old) have the highest incidence of Buruli ulcers, with most lesions on the lower extremities.Image: Extensive Buruli ulcer",Skin,Bacterial Infection of Skin +8b679fd4-ddf7-4292-8db8-815e18215fdd,A 41 year old patient presented with chronic diarrhoea for 3 months. A d-xylose absorption test was ordered to look for:,Carbohydrate malabsorption due to mucosa] disease,Carbohydrate malabsorption due to chronic pancreatitis,Fat malabsorption due to mucosal disease,Fat malabsorption due to chronic pancreatitis,0,single,Ans is A (Carbohydrate malabsorption due to mucosal disease) `The urinary D-xylose test for carbohydrate absorption provides an assessment of proximal small intestine mucosal function.' Urinary D-xylose test: Principle D-xylose is a `pentose. that is completely absorbed almost exclusively in the proximal small intestine and excreted subsequently in the urine. The level of excreted D-xylose in urine is estimated. Any decrease in its level in urine means abnormality in carbohydrate absorption in proximal intestine. Procedure & interpretation 25g D-xylose is given and urine is collected for 5 hrs. An excretion of < 4.5e primarily reflects the presence of duodenal /jejuna) mucosal disease.,Medicine, +937dc386-a681-40b5-891f-3e2a125907cf,Which of the following extraocular muscles does not arise from annulus:,Inferior rectus,Medial rectus,Lateral rectus,Superior oblique,3,single,"The annulus of Zinn, also known as the annular tendon or common tendinous ring, is a ring of fibrous tissue surrounding the optic nerve at its entrance at the apex of the orbit.It can be used to divide the regions of the superior orbital fissure.The aeries surrounding the optic nerve are sometimes called the ""circle of Zinn-Haller"" (CZH).Some sources distinguish between these terms more precisely, with the annulus tendineus communis being the parent structure, divided into two pas:A lower, the ligament or tendon of Zinn, which gives origin to the Rectus inferior, pa of the Rectus internus, and the lower head of origin of the Rectus lateralis.An upper, which gives origin to the Rectus superior, the rest of the Rectus medialis, and the upper head of the Rectus lateralis. This upper band is sometimes termed the superior tendon of Lockwood.The site of origin of the superior oblique muscle is from the lesser wing of sphenoid above the optic canal.",Anatomy, +7e09fed0-c064-482b-802c-77f23bd4a1c2,"Compared to follicular Ca, papillary Ca of thyroid have :a) More male preponderanceb) Bilateralityc) Local recurrence commond) Increased mortalitye) Increased lymph node metastasis.",ab,bce,acd,ace,1,single,,Surgery, +9c65da0f-5b74-4a30-9da0-fbe0a9dee64a,Ptosis is due to:,3rd CN palsy,4th CN palsy,5th CN palsy,6th CN palsy,0,single,"Ans. (a) 3rd CN palsy (oculomotor)Ref.:BDC 6th ed. Vol-III/361-62* Ptosis is drooping of eyelid.* Eyelid is comprised of 4 muscles:Muscles, their innervation and functionMuscleCN innervationFunction* LPSIIIOpens the eyelid* Muller's muscleT1 (NOT a CN; sympathetic nerve)Opens eyelid when LPS tired* FrontalisVIICloses the eyelid* Orbicularis oculiVIICloses the eyelid* CN III palsy leads to drooping of eyelid (Ptosis).* CN VII palsy may also cause ptosis.Also Know*All the extraocular muscles are supplied by CN III except lateral rectus and superior oblique.* Lateral rectus supplied by- CN VI* Superior oblique supplied by- CN IVRemember- LR6; So4",Anatomy,Orbit +07d8e80e-51bc-4550-b39e-aeede6810447,True about isoenzymes is/are,Different km value,Consist of multimeric complex,Have different physical properties,All of these,3,multi,"Consist of multimeric complex & `e' i.e., Have different physical properties + +isozymes are the physically distinct forms of the same enzymes that catalyze the same reaction, and differ from each other structurally, electrophoretically and immunologically. +They differ in their physical properties because of genetically determined difference in amino acid sequence. +They are separated by electrophoresis as they have different electrophoretic mobility. +They have different Kin value. +Isoenzyme of an oligomeric enzyme process differ in combination of its peptide protomer.",Biochemistry, +6dc3ca7d-e70e-4a71-9d19-d8ca18fe74cc,Which of the following is not an action of epinephrine when administered intravenously in a high dose?,Increases liver glycogenolysis,Causes broachiolar constriction,Evokes extrasystoles in the heart,Produces restlessness and anxiety,1,single,,Pharmacology, +5e5a624d-7d19-459a-8462-97deb1ede7d5,Osteoclast has specific receptor for:,Parathyroid hormone,Calcitonin,Thyroxin,Vit D3,1,single,"Calcitonin receptor is expressed in osteoclasts. Calcitonin inhibits bone resorption by inhibiting osteoclasts; and promotes Ca2+ from blood to be deposited on the bone. This decreases blood Ca2+. PTH is a primary endocrine regulator of bone remodeling in adults. The PTH/PTHrP receptor is expressed on osteoblasts, but not on osteoclasts.",Physiology,Endocrine System +968f0797-5a02-45e6-8ddc-2174b37144ee,All are true about precocious puberty except -,Secondary sexual characters before the age of 8 years,Secondary sexual characters before the age of 6 years,Menstruation before the age of 10 years,Most common cause is constitutional,1,multi,"Ans. is 'b' i.e., Secondary sexual characters before the age of 6 years o Precocious puberty is the appearance of appropriate secondary sexual characters before the age of 8 years and occurrence of menstruation before 10 years of chronological age.o It is divided into the following types:A) Gonadotropin dependent precocious puberty/ Central precocious puberty / True precocious puberty:# Most common cause is constitutional, due to early maturation and activation of the hypothalamic - pituitary - gonadal axis.# Both breast and pubic hair development in girls# Pubic hair development and testicular enlargement in boys# Early developing sexual characteristics are ""isosexual"", meaning they are consistent with the child's gender.# Initial spurt in the height followed by premature closure of epiphysis with the result the ultimate height remains stunted.B) Gonadotropin - independent precocious puberty/ Peripheral precocious puberty/Pseudo - precocious puberty:# Exposure to sex steroid hormones that derive from the gonads, the adrenals or the environment.# Further subclassified as isosexual when sexual characters are consistent with gender, and as contrasexual when inconsistent with gender (virilization in girls, or feminization in boys).# Autonomous functional ovarian follicular cysts are the most common cause of gonadotropin - independent precocious puberty in girls.C) Incomplete precocious puberty':# Children with isolated premature thelarche or premature adrenarche. Both usually represent variants of normal pubertal development, but some will progress to complete precocious puberty that may be gonadotropin - dependent or independent.",Gynaecology & Obstetrics,Disorders in Menstruation +57fa1424-b761-4bfd-b26a-5d69ec3c3752,Initiation of nerve impulse occurs at the axon hillock because :,It has a lower threshold than the rest of axon,It is unmyelinated,Neurotransmitter release occurs here,None of the above,0,multi,"A i.e. It has lower threshold than rest of axonAxon hillock is the initial segment of neuron, nerve impulses begin in this initial segment of axon because it has lowest threshold potential.QAxon hillock:- This is a thickened area of cell body from which originates the long fibrous axon.AP is generated in this segment because the membrane at the axon hillock has grater concentration of voltage gated Na+ channels and therefore the threshold for excitation is lower than the rest of neuronsQ",Physiology,